Categories
Harvard Socialism Suggested Reading Syllabus

Harvard. Reading list for Economics of Socialism. Bergson, 1977

The list of readings and final exam for Abram Bergson’s Harvard course “Normative Aspects of Economic Policy” (1960) were posted earlier. In this post Economics in the Rear-view Mirror provides the course outline and assigned readings for his “Economics of Socialism”. I encountered his 1961 book The Real National Income of Soviet Russia Since 1928 in four of my courses (taught by Raymond Powell and John Michael Montias at Yale; Evsey Domar at M.I.T.; and from Bergson himself at Harvard).

_____________________________

HARVARD UNIVERSITY
Department of Economics
Economics 1200:
Economics of Socialism

Spring Term, 1976-77
Professor Bergson

Note
The following will be the principal texts for the course:

Abram Bergson, The Economics of Soviet Planning, Yale, New Haven, Conn., 1964.

Nai-Ruenn Chen and Walter Galenson, The Chinese Economy Under Communism, Aldine, Chicago, 1969

Joel B. Dirlam and James L. Plummer, An Introduction to the Yugoslav Economy, Merrill, Columbus, Ohio, 1973.

Paul R. Gregory and Robert C. Stuart, Soviet Economic Structure and Performance, Harper and Row, New York, 1974.

Note that the Bergson, Gregory and Stuart, and Dirlam and Plummer books are available in paperback.

Items Marked with an asterisk are optional.

I. Introduction
  1. What is Socialism?

“Socialism” (by Daniel Bell), in International Encyclopedia of the Social Sciences, Vol. 14, 1968, pp. 506-516.

Karl Marx, Critique of the Gotha Programme, International Publishers, 1938, pp. 3-23.

V. I. Lenin, State and Revolution, Ch. 5, “The Economic Base of the Withering Away of the State.”

Paul M. Sweezy, “Alternative Conceptions of Socialist Development” (Processed).

Alec Nove, “Market Socialism and Its Critics,” Soviet Studies, July 1972.

II. Comparative Development Strategy
  1. The Soviet Model

Gregory and Stuart, Soviet Economic Structure and Performance, Chs. 1-3, 12 (pp. 417-428 only).

A. Nove, An Economic History of the USSR, London, 1969, Chs. 6-8.

A. Erlich, “Preobrazhenski and the Economics of Soviet Industrialization,” Quarterly Journal of Economics, February 1950.

I.V. Stalin, “On the Grain Front,” “Right Danger,” “Right Deviation,” in Selected Writings, New York, 1942.

  1. Variants

Oleg Hoeffding, “Soviet State Planning and Forced Industrialization as a Model for Asia,” Problems of Communism, Nov.-Dec., 1959; reprinted in F. Holzman, Readings on the Soviet Economy, Chicago, 1962.

Chen and Galenson, The Chinese Economy under Communism, Chs. 1, 2.

A. Eckstein, China’s Economic Development, Ann Arbor, Michigan, 1975, pp. 9-22, 47-51.

See Sweezy under Topic 1.

III. Economic Organization and Planning
  1. Socialist Planning: Contents and Issues

O. Lange “On the Economic Theory of Socialism” including Appendix, in B. Lippincott ed., On the Economic Theory of Socialism, Minneapolis, 1938; New York, 1964.

A. Bergson “Market Socialism Revisited,” Journal of Political Economy, October 1967 (Section on “Cooperative Variant” optional).

W. N. Loucks, Comparative Economic Systems, 7th ed., New York, 1965, pp. 108-120 (5th ed., pp. 98-110; 6th ed., pp. 93-105).

Joan Robinson, An Essay on Marxian Economics, 2nd ed., New York, 1966, pp. 10-28.

Note: As a preliminary to the foregoing readings, you may wish review relevant theoretic foundations in, say, Robert Dorfman, Prices and Markets, New Jersey, 1967, Chs. 7-8.

  1. Centralist Planning in the USSR: The Industrial Enterprise and Collective Farm

Bergson, The Economics of Soviet Planning, Ch. 5 and pp. 287-297; Chs 9 and 10.

J. Berliner The Innovation Decision in Soviet Industry, Cambridge, Mass., 1976, Chs. 14-16.

Gregory and Stuart, Soviet Economic Structure and Performance, Chs. 7 (pp. 232-253), 10.

D. Granick*, “Managerial Incentives in the USSR and in Western Firms,” Journal of Comparative Administration, August 1973.

Emily C. Brown, Soviet Trade Unions and Labor Relations, Cambridge, Mass., 1966, Chs. 7, 9.

E. G. Liberman*, Economic Methods and the Effectiveness of Production, New York, 1973, pp. 21-47.

  1. Centralist Planning in the USSR: Coordination

Bergson, Economics of Soviet Planning, Chs. 1, 3,4, 7, 8,(*) 11.

Liberman*, Economic Methods and the Effectiveness of Production, pp. 75-116.

H. S. Levine, “Pressure and Planning in the Soviet Economy,” in H. Rosovsky, ed., Industrialization in Two Systems, New York 1966; reprinted in M. Bornstein and D.R. Fusfeld, eds., The Soviet Economy, 3rd ed., Homewood, Ill., 1970.

G. Grossman*, “Scarce Capital and Soviet Doctrine,” Quarterly Journal of Economics, August 1953, reprinted in Holzman, Readings.

A. Nove, The Soviet Economy, New York, 1961, Rev. ed., Ch. 3, Ch. 7 (pp. 231-240).

R. W. Campbell, “Marx, Kantorovich, and Novozhilov,” in Slavic Review, October 1961; reprinted in H. Schaffer, The Soviet Economy, New York, 1963; and in George Feiwel, New Currents Soviet-Type Economies: A Reader, Scranton, PA, 1968.

G. Schroeder, “The 1966-67 Soviet Industrial Price Reform,” Soviet Studies, April 1969.

H. Kohler, Welfare and Planning, New York, 1966, pp. 82-95, 102-105.

M. Goldman, “Externalities and the Race for Economic Growth in the USSR: Will the Environment ever Win?” Journal of Political Economy, March/April 1972.

  1. Market Socialism in Hungary and Yugoslavia

Bela Balassa. “The Firm in the New Economic Mechanism in Hungary,” in M. Bornstein, ed. Plan and Market, New Haven, Conn., 1973.

D. Granick, “The Hungarian Economic Reform,” World Politics, April 1973, reprinted in M. Bornstein, ed., Comparative Economic Systems, 3rd ed., Homewood, Ill., 1974.

J. Vanek, The Participatory Economy, Ithaca, New York, 1971, Chs. 2-3.

Dirlam and Plummer, An Introduction to the Yugoslav Economy Chs. 2, 3, 4 (pp. 88-99), 5 (pp. 122-141), 7 (pp. 165-177).

D. D. Milenkovich, Plan and Market in Yugoslav Economic Thought,New Haven, Conn., 1971, pp. 187-211.

D. D. Milenkovich*, “Plan and Market: The Case of Yugoslavia” (Processed).

  1. Planning in China: How Different?

Chen and Galenson, The Chinese Economy Under Communism, Ch. 6

Barry Richman. “Capitalists and Managers in Communist China,” Harvard Business Review, January/February 1967.

D. Perkins, “Industrial Planning and Management,” in A. Eckstein, W. Galenson and T. C. Liu, eds., Economic Trends in Communist China, Chicago, 1968.

Eckstein, China’s Economic Development, Ch. 12.

IV Foreign Economic Relations
  1. Foreign Economic Relations

F. D. Holzman, Foreign Trade Under Central Planning, Cambridge, Mass., 1974, Chs. 2, 6 (analysis of Fig. 6.1, p. 146 and section on foreign trade discrimination, pp. 150-152 are optional).

F. L. Pryor, The Communist Foreign Trade System, Cambridge, Mass., 1963, Chs. 1, 5 (pp. 131-139).

E. A. Hewett, Foreign Trade Prices in the Council for Mutual Economic Assistance, Cambridge, Eng., 1974, Ch. 2.

R. F. Dernberger, “Prices, the Exchange Rate and Economic Efficiency in the Foreign Trade of Communist China,” A. A. Brown and E. Neuberger, eds., International Trade and Central Planning, Berkeley, California, 1968.

V. Performance
  1. Comparative Productivity and Growth

S. Cohn, Economic Development in the Soviet Union, Lexington, Mass., 1970, Chs. 4, 6.

A. Bergson, Planning and Productivity Under Soviet Socialism, New York, 1968 Chs. 1-3.

R. W. Campbell, Soviet Economic Power, 2nd ed. Boston, Mass., 1966, Ch. 6.

A. Bergson “Development Under Two Systems: Comparative Productivity Growth Since 1950,” World Politics, July, 1971; reprinted in Bornstein, Comparative Economic Systems, 3rd ed.

B. Ward, “Capitalism vs. Socialism: A Small Country Version,” in G. Grossman, ed., Essays in Socialism and Planning in Honor of Carl Landauer, Englewood Cliffs, New Jersey.

Chen and Galenson, The Chinese Economy Under Communism, Ch. 9.

Eckstein, China’s Economic Development, Ch. 1.

John G. Gurley, “Capitalist and Maoist Economic Development,” Bulletin of Concerned Asian Scholars, April-July 1970, pp. 42ff.

Reading Period:

Wage Determination and Inequality

Bergson, The Economics of Soviet Planning, Ch. 6.

Bergson, The Structure of Soviet Wages, Cambridge, Mass., 1944, Chs. 2, 13, 14.

M. Matthews*, “Top Incomes in the USSR: Towards a Definition of the Soviet Elite,” Survey, Summer, 1975.

Charles Hoffman, “Work Incentives in Chinese Industry and Agriculture,” in Joint Economic Committee, Congress of the United States, An Economic Profile of Mainland China, Vol. 2, Washington, D.C., February 1967.

Convergence?

J. K. Galbraith, The New Industrial State, Boston, 1967, Ch. XXXV.

Bertram Wolfe, “Russia and the USA: A Challenge to the Convergence Theory” and J.K. Galbraith, “Reply,” American Humanist, September/October 1968.

Peter Wiles, “Convergence: Possibility and Probability” in Balinky et al., Planning and the Market in the USSR, Rutgers, 1967.

Source: Personal copy of Irwin Collier.

Portrait of Abram Bergson. See Paul A. Samuelson, “Abram Bergson, 1914-2003: A Biographical Memoir”, in National Academy of Sciences, Biographical Memoirs, Volume 84 (Washington, D.C.: 2004).

Categories
Exam Questions Harvard Suggested Reading Syllabus

Harvard. Reading List and Final Exam for Games and Strategy. Schelling, 1963

 

Thomas Schelling was hired by the Harvard economics department as a professor in 1958. According to the Harvard course catalogues, he taught the undergraduate course “Games and Strategy” nine times during the 1960’s.  This post provides the syllabus/reading list and final exam for that course from the first term of the 1963-64 academic year.

Materials from Schelling’s course “Economics and National Security” that he taught in 1960 and from his 1970 course “Conflict, Coalition and Strategy” have been transcribed and posted earlier here at Economics in the Rear-view Mirror.

________________________

Course Announcement

Economics 135. Games and Strategy

Half course (fall term). M., W., F., at 10. Professor Schelling

Theories and experimental studies of rational decision in conflict, collusion, coalition, bargaining, collective decision, arbitration, and uncertainty.

Source: Official Register of Harvard University. Vol. LX, No. 21 (September 4, 1963): Faculty of Arts and Sciences. Courses of Instruction for Harvard and Radcliffe 1963-1964, p. 103.

________________________

Economics 135
Games and Strategy
Fall, 1963

Reading Assignments

PART I. COPING WITH AN INTELLIGENT ADVERSARY

  1. Rapoport, Anatol: Fights, Games and Debates, Chapters 7, 8, 9; pages 130-165. (35 pages)
  2. Williams, John D.: The Compleat Strategyst, Chapters 1, 2; pages 1-85, and Chapter 3, pages 86-91 then scan rest of chapter. (91 pages)
  3. Hitch, Charles J. and McKean, Roland: The Economics Defense in the Nuclear Age, Chapter 10, “Incommensurables, Uncertainty, and the Enemy,” pages 182-205. (23 pages)
  4. Read, Thornton: “Strategy for Active Defense,” Papers and Proceedings of the AEA, American Economic Review, Vol. 51, No. 2, May 1961, pp. 465-471.
  5. Alchian, Armen A.: “The Meaning of Utility Measurement,” American Economic Review, Vol. 43 (March 1953) pages 26-50. (25 pages)

(OPTIONAL: R. Duncan Luce and Howard Raiffa, Games and Decisions, Chapters 1-4, pp. 1-87.)

PART II. COERCION AND DETERRENCE

  1. Schelling, Strategy of Conflict, Chapters 1, 2, 5, 7, 8; pages 3-52, 117-161, 175-203. (121 pages)
  2. Ellsberg, Daniel: “The Theory and Practice of Blackmail,” (38 pages) mimeograph
  3. Schelling: “The Threat of Violence in International Affairs,” Proceedings, 57th Annual Meeting, American Society International Law. (INT. 16.8)
  4. Stevens, Carl M.: Strategy and Collective Bargaining Negotiation, chapters 3 and 5, pages 27-56 and 77-96. New York: McGraw Hill, 1963.

PART III. MUTUAL RESTRAINT

  1. Kenneth: Conflict and Defense, Chapters 1, 2, 6, pp. 1-40, 105-122. (58 pages)
  2. Schelling: Chapters 3, 4, 9, 10; Appendix A; pages 53-118, 207-254, 257-266. (121 pages)
  3. Cassady, Ralph, Jr.: “Taxicab Rate War,” Journal of Conflict Resolution, Vol. 1, pages 364-8 (December, 1957). (5 pages)
  4. Valvanis, Stephan: “The Resolution of Conflict When Utilities Interact,” Journal of Conflict Resolution, Vol. 2 (June 1958) pages 156-69. (13 pages)
  5. Rapoport, Chapter 10, pp. 166-79 (14 pages)
  6. Boulding, Chapters 12, 13, pp. 227-73.
  7. Schelling: “War Without Pain and Other Models,” World Politics, XV, (April, 1963) pp. 465-487.

PART IV. COLLECTIVE DECISION AND ARBITRATION

  1. Farguharson, Robin: “Sincerity and Strategy in Voting,” mimeograph (February 5, 1955) (7 pages)
  2. Black, Duncan: “On the Rationale of Group Decision Making,” Journal of Political Economy, Vol. 56 (February, 1948), pages 23-34 (12 pages)
  3. Steinhaus, Hugo: “The Problem of Fair Division,” Econometrica, Vol. 16 (January, 1948), pages 101-109. (9 pages)
  4. Dahl, Robert A.: A Preface to Democratic Theory, Chapter 2, pages 34-60, with special attention to notes 9 and 12, pages 42-43 and 43-44. (26 pages)
  5. Rapoport, Chapter 11, pp. 180-194. (15 pages)
  6. Rapoport, Chapter 12, pages 195-212. (17 pages)

PART V. EXPERIMENTAL GAMES

  1. Flood, Merrill M.: “Some Experimental Games,” Management Science, Vol. 5 (October, 1958) pages 5-26. (22 pages)
  2. Kaplan, Burns, and Quandt: “Theoretical Analysis of the Balance of Power,” Behavioral Science, Vol. 5 (July, 1960), pages 240-52. (12 pages)
  3. Schelling: Chapter 6, pages 162-72. (11 pages)
  4. Rapoport: Chapter 13, pages 213-25. (12 pages)

READING PERIOD

  1. Burns, Arthur L.: “A Graphical Approach to some Problems of the Arms Race,” Journal of Conflict Resolution, Vol. 3, pages 326-42. (16 pages)
  2. Thibaut, John W. and Kelley, Harold H.: The Social Psychology of Groups, Chapter 7, pages 100-125. (26 pages)
  3. Goffman, Irving: “On Face-Work,” Psychiatry: Journal for the Study of Interpersonal Processes, Vol. 18 (August 1955), pp. 213-31.
  4. Twain, Mark, “The Man that Corrupted Hadleyburg,” in The Complete Short Stories of Mark Twain.

Source: Harvard University Archives. Syllabi, course outlines and reading lists in Economics, 1895-2003. Box 8, Folder “Economics, 1963-64”.

________________________

FINAL EXAMINATION
ECONOMICS 135
January 29, 1964

ANSWER ALL FIVE QUESTIONS: The first two questions should take no more than ten or twenty minutes each, allowing at least forty-five minutes each for the last three.

  1. The following entry was submitted to the PUNCH “Toby competition” calling for an “unpleasing codicil to a will,” and received a runner-up award in the issue of July 6, 1960:

To my daughter, Judith Georgina Margaret, I leave my house, land and all my worldly possessions therein on the condition that it should be run as either an hotel, a college for gardeners or a rest-house for disappointed Beatniks.

My cash and capital are to be put into a trust. My widow, three daughters and nine grandchildren will each have an equal share in the trust. No income or capital can be drawn from the trust until the will is contested by a legatee. If this happens, the contesting legatee will lose his share to the others. If the others pay compensation for this loss, all capital will go untied to a charity.

Describe and discuss in game-theoretical terms the arrangement described in the second paragraph. Draw a matrix to represent it. (For purposes of the matrix, you may reduce the number of legatees to two.) Include, with respect to the two-person matrix, any pertinent references to a “solution,” “equilibrium point,” dominant or dominated strategies, or “efficiency” of outcome.

  1. It has been observed that for many people an important criterion in sending or not sending a Christmas card to someone is whether or not they expect to receive one from the person. They would be embarrassed if they received one and had not sent one, but would rather not bother sending one unless they were going to receive one. They might also not wish to embarrass the recipient by sending a card he did not expect and implying he had been negligent. It may not be going too far to suppose that some people, in deciding whether or not to send a card, recognize that the other person, in deciding whether or not to send his card, is wondering whether or not he will get a card.
    Draw a matrix corresponding to this situation, explaining your choice of numerical payoffs, and analyze the situation in familiar fashion.
  2. Two companies, Vitamins, Inc. and Hormones, Inc., sell to groups of potential customers that partially overlap. Some potential consumers of vitamins can meet their needs with hormones, but not all of them; and some potential consumers of hormones can meet their needs with vitamins, but not all of them. Prices are such that the two commodities are pertinent[sic, “perfect”?] substitutes for each other within the overlapping market. Advertizing is the principle form of competition between the two companies. Advertizing also increases, for Vitamins, Inc., sales to those who have no interest in hormones, and similarly for Hormones, Inc.
    The total advertizing budgets for the two companies are fixed by long-term contracts. In the short run they can vary the content of their advertizing. Specifically, the vitamin company can emphasize those uses of vitamins that compete with hormones or those uses that do not. If it emphasizes the uses that do compete, it tends to increase its share of the common market; if it emphasizes the virtues peculiar to vitamins it will increase consumption of vitamins by those who have no interest in hormones but will tend to lose in the market common to both. And similarly for the hormone company. A good deal of research has been done by both companies, leading to advertizing policies that take the rival’s advertizing campaign into account.

V has settled on the following policy:

      1. If H puts less than 20% of its budget into competitive advertizing, V will put none into that form;
      2. If H puts 20% or more into the competitive form, V will put the same percentage into competitive advertizing as H does.

H has arrived at the following policy:

      1. If V puts less than 25% into competitive advertizing, H will put twice that percentage into competitive advertizing;
      2. If V puts 25% or more into competitive advertizing, H will put exactly 50% into that form.

The Problem:

    1. Sketch the “partial equilibrium curves,” and analyze what may happen if each of the two firms simply reacts to what it sees the other doing.
    2. If H gets sophisticated and understands V’s behavior (but V goes on just reacting to what Hdoes), what policy do you expect H to follow, with what result?
    3. If both get sophisticated and realize the nature of their interacting policies, how does your analysis change?
    4. Reinterpret this problem in terms of two countries with fixed defense budgets, allocating their military resources into “offensive” and “defensive” components.
  1. A three-man board composed of A, B, and C, has held hearings on a personnel case involving an officer of the company. This officer was scheduled for promotion but, prior to final action on his promotion, he took a decision that cost the company a good deal of money. The question is whether he should be (1) promoted anyway, (2) denied the promotion, or (3) fired.
    The board has discussed the matter at length and is unable to reach unanimous agreement. In the course of discussion it has become clear to all three of them that their separate opinions are as follows:

A considers the officer to have been a victim of bad luck, not bad judgment, and wants to go ahead and promote him but, failing that, would keep him rather than fire him.
B considers the mistake serious enough to bar promotion altogether; he’d prefer to keep the officer, denying promotion, but would rather fire than promote him
C thinks the man ought to be fired but, in terms of personnel policy and morale, believes the man ought not to be kept unless he is promoted, i.e. that keeping on an officer who has been declared unfit for promotion is even worse than promoting him.

To recapitulate, their preferences among the 3 outcomes are:

Promote Keep Fire
A 1st 2nd 3rd
B 3rd 1st 2nd
C 2nd 3rd 1st

They must proceed to a vote. Voting is by majority. These are the two alternative procedures for voting, and they must first vote on which procedure to use. These alternative procedures are:

    1. Decide first, by majority vote, whether or not he is guilty of a mistake. If (I) he is not found guilty, promote him; if (II) he is found guilty, decide by another majority vote whether to (i) fire him or (ii) to keep him.
    2. By majority vote decide first, as a matter of principle, on the proper course of action if he is guilty — (I) to fire him or (II) to keep him without promotion. Then, once the appropriate penalty has been decided, decide by another majority vote whether he is guilty or not,

(i) promoting him if not guilty, otherwise
(ii) proceeding in accordance with the penalty decided on (I or II) in the first vote.

They must first elect one of the two procedures. They do this, too, by majority vote. They first hold a majority vote to choose procedure 1 or 2; they then vote in accordance with the procedure so selected.

Assume that (a) everyone’s preferences among the three outcomes are fully evident as a result of discussion, (b) everyone is shrewd enough and willing to vote in whatever fashion will attain his own preferences, and assumes everyone else will do the same, (c) voting is silent, by simultaneous ballot, and (d) no “deals” can he made among the three voters as to how they will vote.

The question:

      1. What happens to the officer? Promoted, just kept, or fired?
      2. Which of the two voting procedures, 1 or 2, did they elect to use?
      3. What would have happened to the officer if board-member A had preferred not to promote him?
      4. What might have happened if A and B could make a deal and vote accordingly?
      5. Describe some third majority-vote procedure which if it were used, would lead to the officer’s being kept (pursuant to the board’s preferences in the above table).
  1. Goffman says, “To study face-saving is to study the traffic rules of social interaction. …By face-work I mean to designate the actions taken by a person to make whatever he is doing consistent with face. …Thus poise in one important type of face-work, for through poise the person controls his embarrassment and hence the embarrassment that he and others might have over his embarrassment.”

See how far you can go in treating “poise” and “embarrassment” by a Richardson-process interaction model along the lines of Boulding or Valavanis.

Source: Papers Printed for Midyear Examinations [in] History, History of Religions, …, Economics, …, Naval Science, Air Science (January, 196) in the bound volume Social Sciences, Final Examinations January 1964 (HUC 7000.28, no. 150).

Image Source: Harvard Kennedy School Magazine, Summer 2012.

 

Categories
Exam Questions Harvard Suggested Reading Syllabus

Harvard. Introduction to Quantitative Methods. Joint Economics and Graduate School of Public Administration. Bolton, 1964

 

Roger E. Bolton received his Ph.D. from Harvard in 1964. His dissertation adviser was Otto Eckstein and according to his c.v. (May 2019) Roger Bolton  was an Instructor in Economics and nonresident Tutor in Adams House at Harvard at the time he taught the economics course “Introduction to Quantitative Methods” to students of the Graduate School of Public Administration.

___________________________

From GSPA report

…Dr. Roger E. Bolton, Instructor in Economics, was asked to continue his course, Introduction to Quantitative Methods, which was established particularly for the benefit of students in this School who, without previous mathematical training, needed an introduction to modern methods of economic measurement and projection.

Source: Harvard University. Report of the President of Harvard College 1964-1965, p. 376.

___________________________

Course Announcement

Economics 168. Introduction to Quantitative Methods (Offered jointly with the Graduate School of Public Administration). Half course (fall term). Tu., Th., S., at 9. Dr. Bolton

An introduction to national income accounts, input-output tables, index numbers, capital coefficients, and other methods of economic measurement and projection. This course assumes no previous mathematical or statistical training.

Source: Harvard University, Faculty of Arts and Sciences. Courses of Instruction for Harvard and Radcliffe1964-1965, p. 111.

___________________________

Economics 168
Harvard University
Fall Term, 1964

Dr Roger E. Bolton
Littauer 322

There will be no hour examination. Each student will write a 15 page paper, due about midway in the reading period. The paper is to be some sort of “quantitative” analysis of an important policy problem facing the country of the student, if sufficient data are available for that country. The student will be given considerable latitude in selecting a topic but must confer with the instructor about it in advance. The paper must use extensively some body of statistical data, and include some commentary on the data and their reliability, suitability for the analysis. etc.

In addition to the paper and the final exam, there will be a number of problem sets to be worked during the term. The primary purpose of the problems is learning, not testing, but students’ performance on them will be weighed in determining the final grade. The final exam grade will be weighted 60 per cent, the paper grade 25 per cent, and the problems 15 per cent.

Outline and Readings

Students are expected to do at least some of the suggested readings during the term. There will be no special reading period assignment.

I. Introduction to Important Kinds of Economic Quantities (September 29-October 1)

Ackley, Macroeconomic Theory, pp. 78-88.

Freund and Williams, Modern Business Statistics, 1958, chs. 15-10.

II. Basic Statistical Inference (October 3-8)

Beach, Economic Models, pp. 113-171.

Dixon and Massey, Introduction to Statistical Analysis, ch. 3, pp. 11-23; ch. 5, pp.48-54; ch. 6, pp. 76-82, and all of chs. 8 and 9.

III. National Income and Product: Concept and Measurement (October 10-29)

A. Purposes and uses

Kuznets, Economic Change, Ch. 7.

Kuznets, Six Lectures on Economic Growth, pp. 13-18 and all of Lecture 4.

Suggested: Hitch and McKean, The Economics of Defense in the Nuclear Age, Ch. 3.

B. History

Rosen, National Income, ch. 1

Suggested: Studenski, The Income of Nations, Part 1.

C. Basic Concepts

Harvard Business School, Notes on National Accounting.

Rosen, National Income, chs, 4-5.

Suggested: Ackley, Macroeconomic Theory, chs. 2-3.

D. United States Practices

U. S. Dept. of Commerce, National Income, 1954 ed., pp. 27-60.

U. S. Department of Commerce, Survey of Current Business July 1964 issue, tables giving data on the national accounts.

E. The United Nations System

United Nations Statistical Office, A System of National Accounts and Supporting Tables (Studies in Methods, series F, no. 2, Rev. 2 — second revision).

Suggested:

Organization for European Economic Co-operation, A Standardized System of National Accounts, 1958, pp. 31-49, and Systems of National Accounts in Africa, 1960, pp. 9-63.

Dominion Bureau of Statistics, Canada, National Accounts Income and Expenditure, 1926-1956, pp. 103-176.

Central Statistical Office, Great Britain, National Income Statistics: Sources and Methods, 1956, chs. 1-3.

F. Real Output, Deflation Principles, Various Price Indexes

Ackley, Macroeconomic Theory, pp. 88-99.

U. S. Dept. of Commerce, National Income 1954 ed., pp. 153-158.

U. S. Dept. of Labor, Bureau of Labor Statistics, Bulletin 1168, Techniques of Preparing Major BLS Series, chs. 9-10.

Suggested: U. S. Congress, Joint Economic Committee, Hearings on Government Price Statistics, 87th Congress, 1961, especially pp. 9-78 of the report presented by the National Bureau of Economic Research

G. Other Common Indexes

No assigned readings

H. Intertemporal and International Comparisons

Bornstein, “A Comparison of Soviet and United States National Product,” in U. S. Congress, Joint Economic Committee, Comparisons of the United States and Soviet Economies, 1959.

Suggested: Gilbert and Kravis, An International Comparison of National Products and the Purchasing Power of Currencies.

I. Sources of Data

U. S. Dept. of Commerce, National Income 1954 ed., pp. 68-72, 76-86.

Ruggles and Ruggles, National Income Accounts and Income Analysis, ch. 8 and appendix to ch. 8.

United Nations Statistical Office, Methods of National Income Estimation (Series F, No. 8, of Studies in Methods).

J. Issues in National Income Accounting; Reliability of Data

Rosen, National Income, chs. 8-9.

Schelling, “National Income, 1954 edition,” in Review of Economics and Statistics, November 1955.

Schelling, essay in National Bureau of Economic Research, A Critique of the National Income and Product Accounts.

Jaszi, “The Statistical Foundations of the Gross National Product” in Review of Economics and Statistics, May 1956.

Suggested: Musgrave, The Theory of Public Finance, ch.8

IV. Other Summary Accounts and their Relation to the National Income Accounts (October 31-November 3)

A. The Balance of Payments

Kindleberger, International Economics, Ch. 2

U. S. Department of Commerce, Survey of Current Business, July 1964 issue, tables.

Suggested:

“The Bookkeeping of the Balance of Payments,” Morgan Guaranty Survey, May 1962.

Badger, “The Balance of Payments: A Tool of Economics Analysis,” International Monetary Fund Staff Papers, September 1951.

B. The Government Budget

U. S. Bureau of the Budget, The Budget of the United States, Special Analysis A.

U. S. Dept. of Commerce, Survey of Current Business, July 1964 issue, tables.

Suggested: U. S. Congress, Joint Economic Committee, The Budget as an Economic Document, a report by R. Moor (not the hearings of the same title).

V. Models of National Output Determination (November 5-19)

A. The Distinction between Potential and Actual Output

Schultze, National Income Analysis, ch. 6.

Suggested: Hamberg, Principles of a Growing Economy, ch. 7.

B. Potential Output Models

Kindleberger, Economic Development, ch. 4.

Higgins, Economic Development, 642-653.

Hamberg, Principles of a Growing Economy, ch. 8.

Suggested: Knowles,The Potential Economic Growth of the United States, Study Paper 20 of: U. S. Congress, Joint Economic Committee, Employment, Growth and Price Levels, 1960.

C. Actual Output Models

Hamberg, Principles of a Growing Economy, chs. 9-13.

Schelling, National Income Behavior, chs. 13-14.

Suggested:

Schelling, National Income Behavior, the remainder of book, but especially chs. 9-11.

Polak, “Monetary Analysis of Income Formation and Payments Problems, International Monetary Fund Staff Papers, November 1957.

VI. Input-Output Models (November 21-December 5)

A. Purposes and Uses

Chenery and Clark, Interindustry Economics, ch. 1.

B. Principles

Chenery and Clark, Interindustry Economics, chs. 2,3,5,6.

Suggested: Chenery and Clark, ch. 4.

Stone, Input-Output and National Accounts.

Bruno, Interdependence, Resource Use and Structural Change in Israel, chs. 1-3, Appendices A-1, B-2, and B-3.

C. Applications

Leontief, “The Structure of Development,” in Scientific American, September 1963.

Wonnacott, Canadian-American Interdependence, chs. 1-3, 5-7.

One of these three:

Meyer, “An Input-Output Approach to Evaluating the Influence of Exports on British Industrial Production in the Late 19th Century,” in Explorations in Entrepreneurial History, Volume 8, number 1.

Hirsch, “Interindustry Relations of a Metropolitan Area,” Review of Economics and Statistics, Nov. 1959.

Leontief and Hoffenberg, “The Economic Effects of Disarmament,” Scientific American, April 1961.

Suggested: Wonnacott, the remainder of the assigned book. The other two of the above articles by Meyer, Hirsch, and Leontief and Hoffenberg.

D. Evaluation

Dorfman, “The Nature and Significance of Input-Output,” Review of Economics and Statistics, May 1954.

Seers, “The Role of National Income Estimates in the Statistical Policy of an Underdeveloped Area,” Review of Economic Studies, Vol. 20, pp. 159-168, and his rejoinder in the same journal, Vol. 21, pp. 229-231.

Prest, “Comment” (on Seers), Review of Economic Studies, Vol. 21, pp. 223-228.

Peacock and Dosser, “Input-Output Analysis in an Underdeveloped Country,” Review of Economic Studies, Vol. 25, pp. 21-24.

Suggested: Arrow and Hoffenberg, A Time-Series Analysis of Inter-Industry Demands, especially chs. 1-2, appendix to ch. 2, ch. 3, ch. 7, pp. 117-126, 132-133.

VII. Aspects of Development Planning (December 8-19)

A. General

Chenery and Clark, Interindustry Economics, chs. 7, 9, 10.

Bruno, Interdependence, Resource Use, and Structural Change in Israel chs. 4-6.

Tinbergen, The Design of Development.

Rosenstein-Rodan, ed. Formation and Economic Development, pp. 11-32, 68-82.

Suggested:

Rosenstein-Rodan, ed. remainder of book.

Chenery and Clark, ch. 11.

B. Cases

India, Planning Commission, Third Five-Year Plan, 1961; skim enough to get the flavor.

Pakistan, Planning Commission, The Second Five-Year Plan; skim enough to get the flavor.

Mahalanobis, The Approach of Operational Research to Planning in India, 1955, ch. 1, 3, 4, appendix to ch. 4, chs. 5-7, and the three chapters in Appendix II.

C. Investment Criteria

Chenery, “Comparative Advantage and Development Policy, American Economic Review, March 1961.

Suggested:

Galenson and Leibenstein, “Investment Criteria, Productivity, and Economic Development” Quarterly Journal of Economics. August 1955.

Eckstein, “Investment Criteria for Economic Development and the Theory of Intertemporal Welfare Economics.” Quarterly Journal of Economics, February 1957.

Source: Harvard University Archives. Syllabi, course outlines and reading lists in Economics, 1895-2003. Box 8, Folder “Economics, 1964-1965 (1 of 2)”.

___________________________

Final Examination

ECONOMICS 168
FINAL EXAMINATION
Dr. Bolton
January 26, 1965

DO ALL 5 QUESTIONS.
Read each question carefully before beginning it. Please write legibly

  1. Briefly explain each of these terms: (30 minutes)

a. Net National Income at Factor Cost
b. Unbiased estimate (in statistical inference)
c. Commodity Flow Method
d. Statistical Discrepancy (in national income accounting)
e. “Accounting Prices”
f. Participation Rate

  1. Explain how you would use each of the following numbers in making economic policy decisions: (30 minutes)
    1. The number in row i and column j of an input-output “inverse”
    2. The “standard error of estimate” of a regression estimate of the marginal propensity to import
    3. The amount of planned saving which would occur when actual output is equal to potential output.
    4. The income-elasticity of consumption of a specific product
  2. Do three of the following four short problems: (30 minutes)
    1. Explain what amounts must be added to or subtracted from Gross Domestic Product to calculate Personal Disposable Income
    2. In regard to the change from 1963 to 1964 in a certain country, the income-elasticity of total consumption was 1.10, measuring both income and consumption in current prices. The rate of growth of income, in current prices, was 6 per cent. The consumer price index increased by 5 per cent. What was the rate of growth of real consumption between 1963 and 1964?
    3. Which of the following items are included in Gross National Product as calculated in the in the United States:

(1) State and local government interest
(2) Government purchase of land
(3) The level of inventories at the end of the year
(4) The net profit of a government enterprise
(5) Consumers’ repayment of debt
(6) Government sale of gold, from the monetary stock to foreigners
(7) Pensions paid to former government employees
(8) Purchase of repairs to old consumer durable goods
(9) Receipt by U.S. residents of dividends from foreign corporation
(10) Goods produced on government order but not yet paid for by the government

    1. The following values from the Gross National Product accounts of a certain country are available for a certain year. Calculate the government surplus or deficit for the year.
Personal disposable income 400
Personal consumption 360
Residential construction 60
Inventory investment 10
Business Fixed Investment 30
Exports 40
Imports 35
Depreciation allowances 30
Corporate profits taxes 30
Retained corporate earnings 30
Dividends 20
  1. Write an essay on input-output analysis, including a brief explanation of each of the following points: (45 minutes)
    1. The purposes and assumptions of the analysis
    2. The nature of the variables the analysis predicts
    3. The kinds of forecasts which must be made before the analysis can be used
    4. The differences between “open” and “closed” systems
    5. The way in which total Gross National Product is predicted
    6. The kinds of data needed to apply the analysis
    7. The relevance of input-output analysis to underdeveloped countries
      You may use mathematical notation and examples if you choose, but it is not necessary to do so.
  1. (45 Minutes)

In 1964, actual and potential GNP in the country of Karicutta were both $1,000 million, and the capital stock was $3,000 million. Potential GNP in this country can be exactly predicted by this Cobb-Douglas function:

O = A L3/4 K1/4

It is known that in 1965 the rate of growth of A will be 1 per cent, and the rate of growth of L will be 2 percent. All capital in the country is created by investment by corporations, and information about their investment plans for 1965 is given later in the problem.

Actual GNP can be exactly predicted by this model: (all dollar amounts are in millions)

Consumption = C = 50 + .8(1-t)Y
Investment = I = an exogenous variable
Government Purchases = G = an exogenous variable
Exports = X = an exogenous variable
Imports = M = .03Y
GNP = Y = C + I + G + X – M

In 1964, the variables had the following values:

T= .40
I= 250
G = 200
X = 50

The variable t represents the sum of:

.10 = the share of total GNP which is retained corporate earnings (“undistributed profits”)
.30 = the share of total GNP which is taxes

In 1965 it is known that exports will rise to 60. On the last day of 1964, all corporations in the country made the the following joint announcement to their shareholders: “It is necessary to increase our annual investment to 300 million dollars, and we shall do this beginning in 1965. To finance this added expenditure we will retain more profits and reduce dividends. Assuming that GNP will be 1,000 million in 1965 as it was 1964, we can raise the extra 50 million by raising the share of GNP retained by us from .10 to .15. Therefore, during the year 1965 we shall retain 15 per cent of GNP instead of 10 per cent, and we shall invest 300 million dollars.

What will be potential output in 1965? What must government purchases be in order to make actual output equal to potential? The tax rate is not to be changed. Show your calculations clearly in order to receive partial credit if your final answer is not correct.

Source: Harvard University Archives. Social Sciences. Final Examinations. January 1965 (HUC 7000.28, vol. 157). Papers Printed for Mid-Year Examinations [in] History, History of Religions, …, Economics, …,Naval Science, Air Science, January 1965.

Image Source: Roger E. Bolton in the Williams College Yearbook, Gulielmensian 1977, p. 18.

 

Categories
Chicago Exam Questions Suggested Reading Syllabus

Chicago. Course outline, readings, examination for introduction to econometrics. Marschak, 1949

The following course material was transcribed from copies found in Franco Modigliani’s papers at the Economists’ Papers Archive in the David M. Rubenstein Rare Book & Manuscript Library of Duke University. These items are also available in a scanned .pdf file at the Cowles Foundation website at Yale. Modigliani’s original mimeographed copy is for the most part much more legible than the on-line scanned copy at the Cowles Foundation. This is particularly true for the “terminal examination” questions. Over forty pages of typescript for the lectures are also found in the original Cowles Commission Discussion paper.

More on Jacob Marschak can be found in Robert W. Dimand’s “Keynesian Economics at the Cowles Commission” (Review of Keynesian Studies, vol. 2, 2020, pp. 22-25).

________________________

J. Marschak. INTRODUCTION TO ECONOMETRICS
Economics 314
Spring 1949.

314. Introduction to Econometrics: Statistical testing of economic theories. Numerical estimation of demand and cost functions and other functions occurring in the theory of the firm and household, the theory of markets and the theory of national income. Estimation of economic models. Statistical prediction under conditions of changing economic structure and policy. Prerequisites: Econ 310, 311, 312 or equiv. Win [sic] TuTh 3-4:30; Marschak.

Source: University of Chicago.The College and the Divisions, Sessions of 1948-1949. In Announcements Vol. XLVIII (May 25, 1948) No. 4, p. 250.

________________________

INTRODUCTION TO ECONOMETRICS
20 Lectures given at the University of Chicago in Spring, 1949*

Cowles Commission Discussion Papers, Economics: 266

[*To be used jointly with 24 Lectures (same title) given at the University of Buffalo in Spring, 1948.]

Part I. Non-stochastic economics [11 lectures]

  1. Best policy. Goal variable; non-controlled, controlled, strategic variables.
  2. Exogenous variables and structural parameters. Types of prediction.
  3. Determining the structure from theory and data.
  4. An example.
  5. Econometric “pitfalls” due to disregarded variables or relations. Non-idenifiable structures.
  6. [continued]
  7. The identification, continued
  8. Why does the identification problem arise in non-experimental sciences?
  9. Discussion of earlier problems.
  10. Discussion of earlier problems. [continued]
  11. When need we know the structure?

Part II. Stochastic economics: Population properties [8 lectures]

  1. Joint distributions, non-parametric.
  2. Mid-term examination.
  3. Parameters of joint distributions.
  4. Least-squares property of coefficients of linear regression. Properties of normal distributions.
  5. Exogenous and endogenous variables in stochastic economics.
  6. Identification and determination of structure by the method of reduced form: examples.
  7. More examples.
  8. Motion of an economic variable. Dynamic models. The assumption of independent successive disturbances and its implication.

Part III. Stochastic economics: Sample properties [1 lecture]

  1. Useful properties of certain least squares and maximum likelihood estimators. Obtaining maximum likelihood estimates of structure from those of reduced form.
Recommended reading.
Attached Materials**

J. Marschak, “Economic Structure, Path, Policy and Prediction”

__________, “Statistical Inference from Non-Experimental Observations—an Economic Example”

G. Hildreth, “Problems in the Estimation of Agricultural Production Functions”

[**As far as available.]

*     *     *

READING MATERIAL TO BE USED IN COURSE ON INTRODUCTION TO ECONOMETRICS,
SPRING QUARTER, 1949

  1. Allen, R. G. D., Mathematical Analysis for Economists.
  2. American Economic Association, “Survey of Contemporary Economics” (Blakiston Co., 1949).
  3. Haavelmo, T., “The Probability Approach to Econometrics” (Supplement to Econometrica, 194) .
  4. Haavelmo, T., “Quantitative Research in Agricultural Economics,” Journal of Farm Economics, Vol. 29, No. 4, November, 1947.
  5. Girshick, M. A., and T. Haavelmo, “Statistical Analysis of the Demand for Food,” Econometrica, Vol. 15, No. 2. April, 1947.
  6. Klein, Lawrence R., “The Use of Econometric Models,” Econometrica, April, 1947.
  7. Haavelmo, T., “Methods of Measuring the Marginal Propensity to Consume,” Journal of the American Statistical Association, March, 1947.
  8. Klein, Lawrence R., “A Post-Mortem on Transition Predictions,” Journal of Political Economy, August, 1946.
  9. Marschak, J., L. Hurwicz, Abstracts of papers: Econometrica, April, 1946, pp. 165-170.
  10. Koopmans, T., “Statistical Estimation of Simultaneous Economic Relations,” Journal of the American Statistical Association, Vol. 40, December, 1945.
  11. Marschak, J. and William H. Andrews, “Random Simultaneous Equations and the Theory of Production,” Econometrica, Vol. 12, No. 3-4, July-October, 1944.
  12. Marschak, J., “Money Illusion and the Demand Analysis,” The Review of Economic Statistics, 25, February, 1943.
  13. Marschak, J., “Economic Structure, Path, Policy, and Prediction,” American Economic Review, Vol. 37, May, 1947, pp. 81-84. Lil.
  14. Marschak, J., “Statistical Inference from Non-Experimental Observations,” Econometrica, January, 1948, p. 53.
  15. Hurwicz, L., “Some Problems Arising in Estimating Economic Relationships,” Econometrica, Vol. 15, July, 1947.
  16. Tinbergen, J., “Business Cycles in the U.S.A. 1919-1932” (Statistical Testing of Business-cycle Theories. II), League of Nations, Geneva, 1939.
  17. Koopmans, T., “Measurement without. Theory,” Review of Economic Statistics, 29, August, 1947.

*     *     *

J. Marschak.
INTRODUCTION TO ECONOMETRICS
Economics 314, Spring 1949.

Terminal Examination

Note: Try to answer all 4 problems first, omitting the questions (III) in problems 1 and 4. Answer the questions (III) if time remains.

Problem 1. The quantity x and the price p of a perishable farm product (each measured from its population mean) are determined as in the following model (subscripts indicate time):

(1.1) Demand: xt = αpt + ut

(1.2) Supply: xt = βpt-1 + ut

(1.3) The disturbance ut is not autocorrelated; nor is vt.

Show

(I) How to estimate α for a long time series.

(II) What other structural parameters are present?

(III) (If time remains): How would you estimate those other parameters?

Problem 2. The model of the previous problem is modified as follows:

(2.1) Demand: xt = αpt + ut  (ut not autocorrelated)

(2.2) Price fixation: p_{t}=p^{\ast }_{t} , a level fixed every year by decree.

Show

(I) How to estimate α and σuu?

(II) Is the estimate of α the same as in the previous problem?

Problem 3. National income y, consumption c, and annual (saving) investment i are all measured in dollars of constant purchasing power, and

(3.1) c = αy + β + u ;

(3.2) E(u);

(3.3) i = y – c (an identity);

(3.4) i is exogenous.

(I) Show how to estimate α, β, σuu from a long time series of data on y, c, i.

(II) Suppose y, c, i denote the income, consumption and saving of an individual family which can control its savings but not its income. How does this modification affect the model and the estimation procedure from a time series of family data, or from a survey of a large number of families?

Problem 4. A survey of very large number T of firms belonging to the same industry but located in places with different wage-rates w1, …, wT has been made. The price p of the product is the same for all firms. Wage-rates and price are fixed independently of the firms’ action. The output Xt of each firm depends on labor used only, Nt, according to the formula

(4.1) {X_{t}=B_{t}N^{A}_{t}}C , t = 1, …, T

(the elasticity A being the same for all firms.) Hence,

(4.2) xt = bt + Ant, t = 1, …, T

where the small letters (except for t) stand for the logarithms. Further assume that each firm pushes its output to the point where, apart from a random deviation, the ratio \left( w_{t}/p\right)  \equiv R_{t} equals the labor’s marginal product,

(4.3) Rt = (dXt/dN)⋅Ct, t = 1, …, T

where Ct is a random percentage deviation. Hence

(4.4) Rt = R_{t}=AN^{A-1}_{t}\cdot B_{t}C_{t} , t = 1, …, T

(4.5) rt = a + bt + ct + (A-1)nt, t = 1, …, T,

where again small letters indicate logarithms.

Questions:

(I) How to estimate A?

(II) What other structural parameters are present?

(III) (If time remains): How to estimate those?

Source: Duke University. David M. Rubenstein Rare Book & Manuscript Library. Economists’ Papers Archive. Franco Modigliani Papers, Box T1, Folder “Jacob Marschak’s Courses, 1940-1949.”

Image Source: Carl F. Christ. History of the Cowles Commission, 1932-1952

Categories
Exam Questions Harvard Suggested Reading Syllabus

Harvard. Economics of population growth. Syllabus and examination. Kuznets, 1963

Simon Kuznets was born in Pinsk (Russian Empire, now Belarus) April 30, 1901. He went to secondary school in Rovno and in Kharkov. In 1918–1921 he attended Kharkov Commercial Institute, after which he worked 1921–1922 in the Department of Labor Statistics of South Bureau of Council of Trade Unions. His family emigrated to the United States in 1922.

The Kharkiv National University of Economics in Ukraine was named after Simon Kuznets in 2013.

Previously transcribed and posted here at Economics in the Rear-view Mirror: materials from Simon Kuznets’ 1960-61 Harvard course, Economics 203 “Economic Growth and Comparative Economic Structures”.

____________________________

Course Announcement

Economics 286. Economics of Population Growth
Half course (spring term). Tu., 2-3:30. Professor Kuznets.

A review of long-term trends in growth and structure of population and of their economic implications.

Source: Harvard University. Faculty of Arts and Sciences. Courses of Instruction for Harvard and Radcliffe, 1962-1963. Official Register of Harvard University, Vol. LIX, No. 17 (August 20, 1962), p. 108.

____________________________

HARVARD UNIVERSITY
Department of Economics
Economics 286
Professor Kuznets

Spring Term, 1963

List of Readings (preliminary)

The Malthusian Theory

  1. T. R. Malthus, An Essay on the Principle of Population, 1st edition 1798, reprinted by Macmillan 1929 (or other reprints)
  2. James Bonar, Malthus and His Work, London 1885, reprinted in 1924, (particularly Book I, pp. 1-207)

(For Browsing)

  1. Kenneth Smith, The Malthusian Controversy, London 1951
  2. Harold A. Boner, Hungry Generations, New York 1955

The Low-level Equilibrium Trap

  1. R. R. Nelson, A Theory of the Low-Level Equilibrium Trap, American Economic Review, December 1956, pp. 894-908
  2. Harvey Leibenstein, Economic Backwardness and Economic Growth, New York 1957, Chapter 10, pp. 117-173

Historical Background

  1. United Nations, The Determinants and Consequences of Population Trends, New York 1953, Chapter II, pp. 5-20

Mortality

  1. United Nations, volume listed under (7), Chapter IV, pp. 47-70
  2. George J. Stolnitz, A Century of International Mortality Trends, Population Studies, vol, IX, no. 1, July 1955, pp. 24-55 and vol. X no. 1, July 1956, pp. 17-42

(For Browsing)

  1. Thomas McKeown and R. S. Record, two papers on causes of decline in mortality in England in the 18th and in the 19th centuries, Population Studies, vol. IX, 1955, pp. 119-41 and vol. XVI, 1962, pp. 94-122

Fertility

  1. United Nations, volume listed under (7), Chapter V, pp. 71-97
  2. Frank Lorimer, Culture and Human Fertility, UNESCO 1954, pp. 15-251
  3. Kingsley Davis and Judith Blake, Social Structure and Fertility: An Analytic Framework, Economic Development and Cultural Change, vol. IV, no. 3, April 1956, pp. 211-35
  4. Gary S. Becker, An Economic Analysis of Fertility, Demographic and Economic Change in Developed Countries, Universities-NBER Committee volume, Princeton 1960, pp. 209-240
  5. E. E. Hagen, Population and Economic Growth, American Economic Review June 1959, pp. 310-327

Migration

  1. United Nations, volume listed under (7), Chapter VI, pp. 98-134

Economic Implications

  1. United Nations, volume listed under (7), Chapter XIII, pp. 220-38
  2. Simon Kuznets, Population Change and Aggregate Output, in the volume listed under (6), pp. 324-351
  3. Ansley J. Coale and Edgar M. Hoover, Population Growth and Economic Development in Low Income Countries, Princeton 1958, Part Five, pp. 295-335

Source: Harvard University Archives. Syllabi, course outlines and reading lists in Economics, 1895-2003, Folder “Economics, 1962-1963 (2 of 2)”.

____________________________

HARVARD UNIVERSITY
Economics 286
Examination, May 23, 1963

Please answer six and no more than eight questions, choosing at least one question in each of the four Roman numeral groups.

A detailed outline of an answer is a good substitute for writing out the answer in full,

Please write legibly.

I

  1. What are the connections in the Malthusian theory of population growth among the rate of natural increase of population, the “Iron” law of wages, and the law of diminishing returns? In answering consider birth and death rates separately.
  2. What assumptions are made in the low-level equilibrium trap theories concerning the connection between death rates and per capita income? Concerning the connection between birth rates and per capita income? Discuss the validity of these assumptions.

II

  1. What factors made for significant declines in mortality in the period since the late 18th century? In outlining the groups of factors involved, suggest why the declines were delayed for long periods in the 19th century in many developed countries.
  2. Indicate the differences in magnitudes of decline in mortality among (a) age groups; (b) sex groups; (c) urban and rural population. Select any two of these for a discussion of the factors that may have been responsible for the differences in magnitude of mortality decline.

III

  1. Outline the major trends in the birth rates since late 18th century in: (a) older developed countries of Europe; (b) younger offshoots of Europe overseas; (c) underdeveloped countries. In answering the questions, indicate, in case trends are significant, whether they apply to crude rates alone or also to rates per woman of child-bearing age and per married woman of child-bearing age.
  2. What groups of factors have been suggested to account for the downward trends in birth rates in the developed countries? In outlining these, and the relevant theories, mention also the cross-section differentials in birth rates that provide the empirical base for such theories.
  3. What do the current differences in crude birth rates between developed and underdeveloped countries reflect in the way of different marriage rates, different ages of women at marriage, differential fertility by age of mother and age of father, total number of children in a completed family? Suggest some economic implications of these differences.

IV

  1. What relationship between population trends and economic trend account for much of internal and external migration observed since the late 18th century in and among developed countries? In answering, try to specify as closely as possible the economic and the population trends involved.
  2. What effects on the efficiency of labor can be ascribed to internal (and external) migration in developed countries? Indicate briefly the various types of effect that might be usefully distinguished.
  3. What effects on the pattern of life and consumption of the population can be ascribed to internal (and external) migration in developed countries? Indicate briefly the various type of effect that might be usefully distinguished.

Source: Harvard University. Faculty of Arts and Sciences. Papers Printed for Final Examinations [in] History, History of Religions, … , Economics, …Naval Science, Air Science. June 1963. In bound volume: Social Sciences, Final Examinations, June 1963 (HUC 7000.28, Vol. 147 of 284).

Image Source: Simon Kuznets portrait from 1971 in Wikipedia Commons.

Categories
Harvard Suggested Reading Syllabus

Harvard. Junior Honors Reading Lists. Conrad and Henderson, 1958-1959

 

Before the 1958-59 academic year began, tragedy struck the Harvard economics department. Assistant professor of economics Stefan Valavanis (31 years old) was found shot in his tent near Mt. Olympus. He was to be the lead instructor for the Junior year honors course in economics during the up-coming year. With his loss the course was left to his assistant professor colleagues, Alfred Haskell Conrad and James Mitchell Henderson.

Alfred Haskell Conrad, Ph.D. 1954. The Redistribution of Incomes and the Matrix Multiplier: The Impact of Fiscal Policy on the Distribution of Income in 1950. Advisors: W.W. Leontief and John S. Chipman. (Mathematics Genealogy Project)

James Mitchell Henderson, Ph.D. 1955. The Efficiency of the Coal Industry: An Application of Linear Programming. Advisors: W.W. Leontief and Elizabeth Waterman Gilboy. (Mathematics Genealogy Project)

James M. Henderson was co-author with Richard Quandt of Microeconomic Theory: A Mathematical Approach, the leading graduate microeconomics text in its day and (fun fact) married Anne O. Krueger in July 1981 when they both were professors at the University of Minnesota. The two of them moved on to become professors at Duke University. Henderson died in 1992.

_____________________

Course Announcement

Economics 100. Junior Honors Course

Full course. M., 4-6. Assistant Professors [Stefan] Valavanis (in charge), [Alfred Haskell] Conrad and (spring term) [James Mitchell] Henderson.

Permission required of Assistant Professor Valavanis.

Required course for Economics concentrators who are candidates for honors. This course will deal with the theory of wages and prices; problems of public policy in the fields of Industrial Organization and Labor; the relation between descriptive material and theoretical analysis; methods of testing hypotheses. One two-hour group meeting each week organized as a seminar with papers by students. Additional papers and individual conferences with staff members.

Source: Courses of Instruction Offered by the Faculty of Arts and Sciences 1958-1959. Official Register of Harvard University, Vol. LV, No. 20 (September 3, 1958), p. 89.

_____________________

Course Enrollment

[Economics] 100. Junior Honors Course. Assistant Professors Conrad and (S) Henderson. Full course.

(F) Total 49: 3 Seniors, 40 Juniors, 3 Sophomores, 3 Radcliffe
(S) Total 46: 4 Seniors, 37 Juniors, 2 Sophomores, 3 Radcliffe

Source: Harvard University. Report of the President of Harvard College 1958-59, p. 70.

_____________________

Final Reading List and Course Outline
Economies 100
Fall, 1958

I. Introduction: Some background and material on the nature and setting of American industrial enterprise. The market mechanism.

W. Adams (ed.), The Structure of American Industry, Chs. 5, 6, 7, 11, 13.

A.B. Jack, “The Channels of Distribution for an Innovation,” Explorations in Entrepreneurial History, IX, p. 113.

K.E. Boulding, Economic Analysis, Part I (any edition).

II. The Empirical Measurement of Economic Functions.

M.J. Moroney, Facts from Figures, Ch. 2, 4; (probability and central tendency).

R.E. Freund, Modern Elementary Statistics, Ch. 12, 13.

Jan Tinbergen, Econometrics, Chs. 1, 2, 5.

R. G. D. Allen, Mathematical Analysis for Economists, Ch. 2; Ch. 6 (recommended) (functions and derivatives).

III. Factors Determining Industry Structure.

A. Production Theory

E. A. G. Robinson, The Structure of Competitive Industry, Chs. 1-5.

T. Scitovsky, Welfare and Competition, Chs. 6,7.

V. E. Smith, “The Statistical Production Function,” Quarterly Journal of Economics, 1944-45, pp. 543-62.

R. G. Bressler, Jr. “Research Determination of Economies of Scale,” Journal of Farm Economics, 1945, p. 526.

B. Cost Curves

Committee on Price Determination for the Conference on Price Research, Cost Behavior and Price Policy, pp. 80-115, 291-301, 219-263, 321-329.

J. Viner, “Cost Curves and Supply Curves,” Readings in Price Theory. Ch. 10, p. 198.

M. Colberg, et al, Business Economics, Ch. 4, 5.

C. Barriers to entry other than economies of scale

J. Bain, Barriers to New Competition, Ch. 6.

IV. Markets and Pricing:

A. Indifference Curves and Consumer Theory

George J. Stigler, The Theory of Price (any edition) Ch. 5.

Scitovsky, Welfare and Competition, Chs. 2, 3, 4.

Boulding, Economic Analysis, Ch. 34 (revised edition).

B. Pure Competition and Pure Monopoly

E. H. Chamberlin, Theory of Monopolistic Competition, Chs. 1, 2.

T. Scitovsky, Welfare and Competition, Chs. 8, 16.

C. Oligopoly

Chamberlin, Monopolistic Competition, Chs. 3, 4, 5.

Readings in Price Theory, Chs. 20, 21.

T. Scitovsky, Welfare and Competition, Ch. 20.

R. M. Alt, “Statistical Measurement of Price Flexibility,” Quarterly Journal of Economics, 1949, p. 92.

J. Bain, Barriers, Ch. 7.

J. Vanek, “The Nature of Equilibrium in Monopolistic Competition,” Mimeographed.

V. Investment Theory

A. An outline of formal theory

Samuelson, Economics, Ch. 29.

Boulding, Chs. 35-37 (revised edition).

Joel Dean, Capital Budgeting, Chs. 1-3.

J. M. Clark, “Business Acceleration and the Law of Demand,” Readings in Business Cycle Theory.

J. Meyer and E. Kuh, The Investment Decision, Ch. 11 (2).

B. Uncertainty

R. Weckstein, “On the Use of the Theory of Probability in Economics,” Review of Economics and Statistics, 1953, pp. 191-198.

Ward Edwards, “The Theory of Decision Making,” Psychological Bulletin, 1954.

A. G. Hart, Anticipation, Uncertainty, and Dynamic Planning.

C. Financial Considerations

J. Meyer and E. Kuh, The Investment Decision, Chs. 9, 12.

R. Mack, The Flow of Business Funds and Consumer Purchasing Power, Ch. VIII.

W. Heller, “The Anatomy of Investment Decisions,” Harvard Business Review, March 1951.

Fortune, “The Fine Art of Raising Capital,” u, July 1956 and ”How Much Can Business Borrow?” June 1956.

D. The Influence of Market Structure on the Investment Decision and Innovation.

W. Fellner,” The Influence of Market Structure on Technological Progress,” Quarterly Journal of Economics, November 1951, pp. 556-577.

C. Kaysen, “A Dynamic Aspect of the Monopoly Problem,” Review of Economics and Statistics, May 1949, pp. 109-113.

Schumpeter, Capitalism, Socialism and Democracy, Ch. VIIl, pp. 87-106.

_____________________

Economics 100
Reading List
Spring, 1959

I. General Equilibrium.

E. Phelps Brown, The Framework of the Pricing System, Chaps. II-IV.

F. Zeuthen, Economic Theory and Method, Chaps. 10-12 and pp. 194-197.

J. R. Hicks, Value and Capital, Part II.

W. W. Leontief, “Input-Output Economics,” Scientific American, October, 1951.

W. W. Leontief, “Input-Output Analysis and the General Equilibrium Theory,” in T. Barna, ed., The Structural Interdependence of the Economy, pp. 42-49.

Selected Applications

(The material listed under this heading may be used as the basis for tutorial papers and discussion. Specific assignments will be made by the instructor.)

W. W. Leontief, “Factor Proportions and the Structure of American Trade,” Review of Economics and Statistics, Nov., 1956.

R. S. Eckaus, “The Factor Proportions Problem in Underdeveloped Areas,” American Economic Review, September, 1955.

II. Welfare Economics.

T. Scitovsky, Welfare and Competition, Chaps. 4, 8, 16. (for review).

J. de V. Graaff, Theoretical Welfare Economics. Chaps. I-V, X, XI. (omit the appendices)

K. Boulding, “Welfare Economics,” in Survey of Contemporary Economics, Vol. II, B. F. Haley, ed.

A. P. Lerner, The Economics of Control, Chaps. 3, 4.

Selected Applications

M. Friedman, in R. Solo, ed., Economics and the Public Interest, Chap. 9.

W. S. Vickey, The Revision of the Rapid Transit Fare Structure of the City of New York, (hectographed; in Littauer Library).

J. V. Krutilla and O. Eckstein, Multi-Purpose River Development, 1958.

N. Kaldor, The Expenditure Tax.

III. Macro ModelsCycles, Growth, Money.

G. Haberler, Prosperity and Depression, Chaps. 2, 3, 5.

A. Hansen, Business Cycles and National Income, Part Il.

A. Hansen, Monetary Theory and Fiscal Policy, Chaps. 3-6.

J. S. Duesenberry, Business Cycles and Economic Growth, Chaps. 2-5, 9-12.

C. Christ, “Aggregate Econometric Models,” American Economic Review, June, 1956

H. Makower and J. Marschak, “Assets, Prices and Monetary Theory,” in G. Stigler and K. Boulding, eds., Readings in Price Theory. Chap. 14.

J. Gurley and E. Shaw, “Financial Aspects of Economic Development,” American Economic Review, September, 1955.

Selected applications from the fields of wage and price policy, inflation, international trade and development will be assigned by the instructors.

Source: Harvard University Archives. Harvard University: Syllabi, course outlines and reading lists in Economics, 1895-2003. Box 7, Folder “Economics, 1958-59 (1 of 2).”

Image Sources:

Left. 1959 Alfred Haksell Conrad, John Simon Guggeheim Memorial Foundation.
Right:  1959, James M. Henderson, University of Minnesota Archives.

Categories
Harvard Syllabus

Harvard. Reading lists for Aggregate Economic Theory, Dorfman. 1962

 

 

“Macro-economics” was explicitly named in the course description for the Harvard undergraduate economics tutorial in 1962-63. However, not a single course included “macroeconomics” in its title. Instead graduate students were treated to “aggregate economic theory”, an early and one might argue more felicitous name than “macroeconomics”.  This post provides the reading list for Robert Dorfman’s aggregate economic theory course. During the second term of 1958-1959 the same course content was taught by Dorfman as “Economics 241. Money and Banking”.

_________________________

Course Announcement

Economics 241. Aggregate Economic Theory

Half course (fall term). M., W., (F.), at 12. Professor Dorfman.

Source: Harvard University, Faculty of Arts and Sciences. Courses of Instruction for Harvard and Radcliffe, 1962-1963, p. 106.

_________________________

Fall, 1962

HARVARD UNIVERSITY
Department of Economics

Economics 241
READING LIST NO. 1

TEXTS

J. M. Keynes. The General Theory of Employment, Interest and Money.

American Economic Association, Readings in Business Cycle Theory.

Also recommended:

Alvin H. Hansen, A Guide to Keynes

Introductory Material

A. P. Lerner, “The General Theory (1),” S.E. Harris, ed., The New Economics, Ch. 11.

L. Tarshis, “An Exposition of Keynesian Economics,” R.V. Clemence, ed., Readings in Economic Analysis, Vol. I, pp. 197-208.

Gardner Ackley, Macroeconomic Theory, Chs. II, III, IV.

U.S. Department of Commerce, National Income, 1954 Edition (Supplement to the Survey of Current Business), pp. 27-60 and skim the rest.

T.C. Schelling, “National Income, 1954 Edition,” Rev. of Econ. and Stat., XXXVII, 321-335 (November 1955).

Consumption

J.M. Keynes, General Theory. Book III.

Robert Ferber, “Research on Household Behavior,” Amer. Econ. Rev., LII, 19-63 (March 1962) .

Irwin Friend, Individuals’ Saving, Ch. 8

J.S Duesenberry, Income, Saving and the Theory of Consumer Behavior, Ch. 3.

M. Friedman, A Theory of the Consumption Function, Ch. 9, at least.

G. Haberler, “My. Keynes’ Theory of the Multiplier,” Readings in Business Cycle Theory, Ch. 9.

Fritz Machlup, “Period Analysis and Multiplier Theory,” Readings in Business Cycle Theory, ch. 10.

Investment

J.M. Keynes, General Theory, Chs. 11, 12, 16.

I. Fisher, Theory of Interest, Chs. 5-11.

A. A. Alchian, “The Rate of Interest, Fisher’s Rate of Return over Costs and Keynes’ Internal Rate of Return,” Amer. Econ. Rev., X, 938-943 (December 1955).

J.R. Meyer and E. Kuh, The Investment Decision. Chs. 2, 12.

J.S.  Duesenberry, Business Cycles and Economic Growth, Chs. 3, 5.

F. Modigliani and M.H. Miller, “The Cost of Capital, Corporation Finance and the Theory of Investment,” Amer. Econ. Rev. XLVIII, 261-297 (June 1958).

J.M. Clark, “Business Acceleration and the Law of Demand,” Readings in Business Cycle Theory, Ch. 11.

 

 

Fall, 1962

HARVARD UNIVERSITY
Department of Economics

Economics 241
READING LIST NO. 2

Interest Theory

J. M. Keynes, General Theory. Chs. 13, 14, 15, 17.

G.L.S. Shackle, “Recent Theories Concerning the Nature and Role of Interest,” Economic Journal, 71 (June 1961), 209-254.

A.P. Lerner in S.E. Harris, ed, The New Economics, Chs. 45, 46.

B. Ohlin, “Some Notes on the Stockholm Theory of Saving and Investment,” Readings in Business Cycle Theory, Ch. 5

F. A. Lutz, “The Outcome of the Saving-Investment Discussion,” ibid., Ch. 6.

W. Fellner and H.M. Somers, “Alternative Monetary Approaches to Interest Theory,” Rev. of Ec. And Stat., Feb, 1941.

T. Wilson and P.S.W. Andrews, eds., Oxford Studies in the Price Mechanism, Ch. 1.

R. W. Clower, “Productivity, Thrift, and the Rate of Interest,” Economic Journal, March 1954.

Monetary Theory

Irving Fisher, The Purchasing Power of Money, Chs. 2, 3, 8.

Alfred Marshall, “Minutes of Evidence before the Royal Commission on the Values of Gold and Silver,” Questions 9629-9664 (pp. 34-46), Question 9686 (pp. 51-52).

J.M. Keynes, A Tract on Monetary Reform, pp. 74-87.

A.C. Pigou, “The Value of Money,” in F. A, Lutz and L.W. Mints, eds., Readings in Monetary Theory, Ch. 10

W.F. Crick, “The Genesis of Bank Deposits,” ibid., Ch. 4.

H.S. Ellis, “Some Fundamentals in the Theory of Velocity,” ibid., Ch. 7.

Milton Friedman, “The Quantity Theory of Money—A Restatement,” in M. Friedman, ed., Studies in the Quantity Theory of Money, pp. 3-21.

H. Johnson, “Monetary Theory and Policy,” Am. Ec. Rev., June 1962.

W.J. Baumol, “The Transactions Demand for Cash, Quarterly Journ. of Econ., November 1952.

 

 

Fall, 1962

HARVARD UNIVERSITY
Department of Economics

Economics 241
READING LIST NO. 3

Synthesis of Aggregative Economics

J.M. Keynes, General Theory: Chs. 18, 19, 21.

Franco Modigliani, “Liquidity Preference and the Theory of Interest and Money” in Readings in. Monetary Theory, Ch. 11.

J.R. Hicks, “Mr. Keynes and the ‘Classics’” in Readings in the Theory of Income Distribution, Ch. 24.

A.C. Pigou, “The Classical Stationary State,” Economic Journal, December 1943.

Don Patinkin, “Price Flexibility and Full Employment,” American Economic Review, September 1948.

P.A. Samuelson, “The Simple Mathematics of Income Determination,” in Income, Employment and Public Policy (New York: 1948), 133-155.

D.B. Suite, “Forecasting and Analysis with an Econometric Model.” American Economic Review, March 1962.

Marc Nerlove, “A Quarterly Econometric Model for the United Kingdom,” American Economic Review, March 1962.

Aggregative Models of Economic Growth

R.P. Harrod, Towards a Dynamic Economies, Lecture 3.

E.D. Domar, Essays in the Theory of Economic Growth, Chs. 3-5.

Robert Solow, “A Contribution to the Theory of Economic Growth,” Quarterly Journal of Economics, February 1956.

W.J. Baumol, Economic Dynamics, Ch. 4.

READING PERIOD ASSIGNMENT.

J. M. Keyes, The General Theory of Employment, Interest, and Money, entire.

J.G. Gurley and E.S. Shaw, Money in a Theory of Finance.

Source: Harvard University Archives. Syllabi, course outlines and reading lists in Economics, 1895-2003, Box 8, Folder “Economics, 1962-1963 (1 of 2)”.

Categories
Exam Questions Harvard Socialism Suggested Reading Syllabus

Harvard. The Soviet Economy, course outline and final exam. Herbert Levine, 1963

 

Herbert Levine was trained in economics and Russian studies at Harvard before going off to lifetime employment at the University of Pennsylvania. He returned to Harvard in the fall term of 1963 to cover the Soviet economy class for Abram Bergson who was on leave at the Center for Advanced Study of the Behavioral Sciences at Stanford in 1963-64.

 Along with a younger economist from the University of Texas, Ed Hewett, Levine championed my application to the International Research and Exchanges Board (IREX) for a research exchange fellowship in the German Democratic Republic back in the late 1970s. He was a mentor to many other young scholars working on the economies of the Soviet Union and Eastern Europe. I last saw him in November 2003 at an economics workshop at Harvard where drafts of papers were presented that would later be published in a special issue edited by Paul R. Gregory and Marshall Goldman in honor of Abram Bergson (Comparative Economic Studies, 2005).

_____________________________

Obituary in The Daily Pennsylvanian

Former economics prof. leaves a legacy
47-year teacher is remembered fondly for his compassion and challenging courses

https://www.thedp.com/article/2007/06/former-economics-prof-leaves-a-legacy

By Alissa Eisenberg and Alissa Eisenber 06/14/07

Herbert Levine, Economics professor at Penn from 1960 to 2006, died Sunday, succumbing to complications from leg surgery after battling prostate cancer for the past 15 years.

Levine was 78.

Receiving his B.A. [1950], M.A. [1952] and Ph.D. [1961] degrees from Harvard University, Levine specialized in Soviet economics and his insights were “in demand during the period leading up to the dissolution of the Soviet system,” according to a written statement by fellow Economics professor Lawrence Klein.

Levine published several articles on his area of expertise, yet never failed to acknowledge the importance of the broader study of economics.

Winning several awards for excellent teaching at Penn including the Lindback Foundation Award for Distinguished Teaching and the Kravis Prize for Distinction in Undergraduate teaching, Levine was highly regarded among students.

“Econ 1 is large, but [my dad] would call on people by name, he just taught that way and people cared for his courses,” said daughter and College alumna Jan Levine.

Former student and 1964 College alumnus Ted Kozloff echoed Levine’s revere for her father.

“Herb was a seminal figure in my education,” Kozloff said. “There are maybe one or two teachers in my lifetime that had an effect like Herb. . He enjoyed enormous popularity and there was enormous respect for him.”

And that respect remained prominent over his 47-year career at Penn.

Levine was Elizabeth Goldstein’s dissertation advisor in 1982, and she said he was “the most fabulous adviser anybody could ask for.”

Goldstein added, “He was rigorious but understanding and had an amazing gift for being able to guide people through very difficult and high-level economic theory.”

Many former students also noted his warmth and devotion to his personal life in addition to academics.

“Many people excel in their careers and forget their personal life, but Herb didn’t,” said former student Edward LaPuma.

Levine’s funeral was scheduled for this morning in Trevose, Pa.

He is survived by his wife Helene Levine, daughters Jan and Judith Levine, sister Myra Heller and three grandchildren. His son, Jonathon, predeceased him.

Obituary in the University of Pennsylvania Almanac,
Vol. 54, No. 1. July 17, 2007

https://almanac.upenn.edu/archive/volumes/v54/n01/obit.html

Dr. Herbert S. Levine, professor emeritus of economics and expert on Soviet and post-Soviet Russia, died on June 10, after complications from surgery from a broken leg during the end of a fifteen-year battle with prostate cancer; he was 78.

Dr. Levine completed his postsecondary education at Harvard, earning an undergraduate degree in economics in 1950, followed by a master degree in Russian studies two years later. He also earned a doctorate from Harvard in 1961, writing his dissertation on the economic performance of the USSR, which earned important recognition of his research by winning the prestigious David A. Wells Prize.

Dr. Levine joined Penn’s faculty in 1960 as an assistant professor of economics. He studied the controlled economy of the USSR, in close touch with other members of a research center at Harvard University. He was promoted to professor in 1969. In addition to his teaching duties, Dr. Levine served as chairman of the graduate group in economics and as co-director of the Lauder Institute. After a 47-year career at Penn he retired in 2006.

His unusual abilities in presenting modern political economy to undergraduates resulted in him being awarded faculty prizes for his teaching including the Irving B. Kravis Prize for Distinction in Undergraduate Teaching (1988 and 1991) and the Lindback Award for Distinguished Teaching.

Dr. Levine is survived by his wife, Helene; two daughters, Jan Levine, and Judith Levine and their husbands Michael Zuckerman and Edward Sobel; their grandchildren, Rachel Zuckerman, Joshua Zuckerman and Julia Sobel; and his sister, Myra Heller and brother-in-law Jack Heller.

_____________________________

Harvard Course Announcement Fall Term, 1963

Economics 133. The Economy of Soviet Russia (Offered jointly with the Committee on Regional Studies).

Half course (fall term). M., W., (F.) at 9. Professor Levine (University of Pennsylvania).

Economic development under the five-year plans: the rate of economic growth: structural changes; conditioning factors. Planning principles and procedures.

Source: Harvard University, Faculty of Arts and Sciences. Courses of Instruction for Harvard and Radcliffe, 1963-1964, p. 103.

_____________________________

ECONOMICS 133, Fall Term, 1963-1964
The Economy of Soviet Russia
H. S. Levine

Course Outline and Reading List.

Books to be Purchased:

Dobb: M. Dobb, Soviet, Economic Development Since 1917, International Publishers, 1948.

Readings: Readings on the Soviet Economy, F. Holzman (ed.), Rand McNally, 1961.

Part One

I. Historical Overview

A. Pre-Revolution
(General Historical Background: D. Treadgold, Twentieth Century Russia, Rand McNally, 1959, Chapters 1-5, 8.

1. Bowden, Karpovich and Usher, An Economic History of Europe Since 1250, Chapters 14 and 29.

2. G. T. Robinson, Rural Russia Under the Old Regime, Chapters 6, 7, 11.

3. A. Gerschenkron, “The Rate of Industrial Growth In Russia Since 1885,” Part 1, The Journal of Economic History, Supplement VII, 1947, pp. 144-157 (only).

4. Supplementary Readings:

a) J. Blum, Lord and Peasant in Russia, Chapters 24, 26, 27.

b) Robinson, Chapters 5, 12.

c) P. I. Lyashchenko, History of the National Economy of Russia; on agriculture: Chapters 21, 23, 36; on industry: Chapters 25, 26, 32-34.

d) T. Von Laue, Sergei Witte and the Industrialization of Russia

e) A. Gerschenkron, “Russia: Patterns and Problems of Economic Development, 1861-1958, » in his Economic Backwardness in Historical Perspective (postpone Part Ill). This essay also appears in C. Black (ed.), The Transformation of Russian Society.

B. The Revolution, War Communism, and the New Economic Policy (1917-1927)
(General Historical Background: Treadgold, Chapters 9-14.

1. Dobb, Chapters 3-7.

C. The Industrialization Debates, Collectivization and the Beginning of the Plan Era
(General Historical Background: Treadgold, Chapter 17.

1. Dobb, Chapters 8 and 9

2. A. Erlich, “Preobrazhenski and the Economics of Soviet Industrialization,” Quarterly Journal of Economics, Feb. 1950, pp. 57-88

3. Readings: N. Jasny, “Early Kolkhozy and the Big Drive”

4. M. Fainsod, Smolensk Under Soviet Rule, pp. 242-258

5. Supplementary Reading:

a) Readings: A. Erlich, “Stalin’s Views on Soviet Economic Development”.

D. From the First Five Year Plan to the End of the War (1928-1945)
(General Historical Background: Treadgold, Chapter 18

1. Dobb, Chapter 10

2. N. Jasny, Soviet Industrialization 1928-1952, pp. 109-114

3. Dobb, Chapters 11, 12 and pp. 453-454

4 Jasny, Soviet Industrialization, pp. 177-187

E. Post-War to the Present
(General Historical Background: Treadgold, Chapter 25

1. Jasny, Soviet Industrialization, pp. 235-256

2. G. Grossman, “The Soviet Economy,” Problems of Communism, XIl:2, (Mar-Apr. 1963) pp. 32-40

3. Supplementary Readings:

a) O. Hoeffding, “Substance and Shadow in the Soviet Seven Year Plan,” Foreign Affairs, April, 1959

b) H. Levine, “The New Seven Year Plan,” The New Leader, May 25 and June 1, 1959,

II. Soviet Economic Growth

A. Problems of Measuring Growth

1. The Soviet Statistical System

a) G. Grossman, Soviet Statistics of Physical Output of Industrial Commodities, pp. 1-10, 22-46

2. Reliability of Soviet Statistics

a) Readings: A. Bergson, “Reliability and Usability of Soviet Statistics: A Summary Appraisal”

b) L. Turgeon, “On the Reliability of Soviet Statistics,” Review of Economics and Statistics, February 1952, pp. 75-6.

c) Grossman, Soviet Statistics, pp. 123-134.

3. Interpretation of Data

a) Readings: A. Bergson, “The Adjusted Factor Cost Standard of National Income Valuation.”

b) Readings: A. Nove, “1926/27 and All That.”

c) A. Gerschenkron, A Dollar Index of Soviet Machinery Output, pp. 47-58

d) R. Moorsteen, “On Measuring Productive Potential and Relative Efficiency,” Quarterly Journal of Economics, August 1961, pp. 451-467

e) Supplementary Readings

1) J. R. Hicks, “The Valuation of the Social Income,” Economica, May 1940, pp. 105-124

2) P. Samuelson, “Evaluation of Real National Income,” Oxford Economic Papers, January 1950, pp. 1-29.

3) A. Becker, “Comparisons of US and USSR National Output: Some Rules of the Game,”World Politics, October 1960, pp. 99-111.

B. What Has Been Accomplished?

1. National Income

a) A. Bergson, “National Income,” in A. Bergson and S. Kuznets (eds.), Economic Trends in the Soviet Union, pp. 1-16 (only).

b) S. Cohn, “The Gross National Product in the Soviet Union,” in Joint Economic Committee, Dimensions of Soviet Economic Power, pp. 69-77.

c) Supplementary Reading

1) A. Bergson, The Real National Income of Soviet Russia Since 1928, Chapters 13 and 14.

2. Industry

a) Readings: N. Kaplan and R. Moorsteen, “An Index of Soviet Industrial Growth”

b) R. Greenslade and P. Wallace, “Industrial Production in the USSR,” in Dimensions, pp. 119-130

c) Supplementary Reading

1) R. Powell, “Industrial Production,” in Trends, pp. 150-176

3. Agriculture

a) D. G. Johnson, “Agricultural Production”, in Trends, pp. 203-214

b) J. Willet, “The Recent Record in Agricultural Production,” in Dimensions, pp. 95-100.

4.  Consumption

a) J. Chapman, “Consumption,” in Trends, pp. 235-270

b) Supplementary Reading

1) R. Golden, “Recent Trends in Soviet Personal Income and Consumption,” in Dimensions, pp. 347-366.

C. Analysis of Growth

1. H. Schwartz, Russia’s Soviet Economy (2nd and Revised Editions), pp. 1-26

2. W. Eason, “Labor Force,” in Trends, pp. 38-93

3. N. Kaplan, “Capital Formation and Allocation,” in A. Bergson (ed.), Soviet Economic Growth, pp. 37-80

4. A. Bergson, “National Income,” in Trends, pp. 17-35.

5. Supplementary Readings

a) F. Seton, “Production Functions in Soviet Industry,” American Economic Review, May 1959, pp. 1-14

b) R. Campbell, Soviet Economic Power, Chapter 4

c) N. DeWitte, “Education and the Development of Human Resources,” in Dimensions, pp. 233-268

d) Readings: “Forced Labor in the Soviet Union”.

Ill. The Operation of the Soviet Economy

A. General Operating Framework and Principles

1. P. Cook, “The Administration and Distribution of Soviet Industry,” in Dimensions, pp. 183-210.

2. A. Nove, The Soviet Economy, Chapter 1.

3. W. Loucks, Comparative Economic Systems (6th Edition), pp. 444-453.

4. Supplementary Reading

a) G. Grossman, “The Structure and Organization of the Soviet Economy,” Slavic Review, June 1962, pp. 203-222.

B. Planning

1. O. Lange, “On the Economic Theory of Socialism” in B. Lippincott (ed.), O. Lange and F. Taylor, On the Economic Theory of Socialism.

2. Schwartz, Chapter V

3. Readings: H. Levine, “The Centralized Planning of Supply in Soviet Industry.”

4. Readings: G. Grossman, “Scarce Capital and Soviet Doctrine”

5. Readings: “On the Problem of Determining the Economic Effectiveness of Capital Investments.”

6. Supplementary Readings

a) J. Drewnowski, “The Economic Theory of Socialism: A Suggestion for Reconsideration,” Journal of Political Economy, August 1961, pp. 341-354

b) Dobb, Chapter 1

c) H. Hunter, “Optimum Tautness in Developmental Planning,” Economic Development and Cultural Change, July 1961, Part l, pp. 561-572.

d) R. Campbell, Soviet Economic Power, Chapter 5

e) I. Yevenko, Planning in the USSR, (Foreign Languages Publishing House, Moscow), esp. Chapter III.

f) Readings: D. Granick, “An Organizational Model of Soviet Industrial Planning.”

C. The Firm and Problems of Industrial Administration

1. Readings: J. Berliner, “The Informal Organization of the Soviet Firm.”

2. Readings: A. Nove, “The Problem of Success Indicators in Soviet Industry.”

3. “The Liberman Proposals”

a) Current Digest of the Soviet Press, XIV:36 (Oct. 3, 1962), pp. 13-15

b) Harry Schafer, “Ills and Remedies,” Problems of Communism, May-June 1963, pp. 18-26.

4. A. Nove, “The Soviet Industrial Reorganization,” Problems of Communism, Nov.-Dec. 1957, pp 19-25.

5. H. Levine, “Recent Developments in Soviet Planning,” in Dimensions, 1st Section, pp. 167-185.

6. P. Cook, “Party, State and Economic Reorganization in the USSR.” The ASTE Bulletin, V:l (Winter 1963), pp. 2-11

7. Supplementary Readings

a) Readings: J. Berliner, “Managerial Incentives and Decisionmaking.”

b) G. Grossman, “Soviet Growth: Routine, Inertia, and Pressure,” American Economic Review, May 1960, pp. 62-72.

c) Readings: O. Hoeffding, “The Soviet Industrial Reorganization of 1957.”

d) M. Goldman, “Economic Controversy in the Soviet Union,” Foreign Affairs, 1963:3, pp. 498-512.

e) A. Nove, “The Liberman Proposals,” Survey, April 1963, pp. 112-118.

D. Prices

1. M. Bornstein, “The Soviet Price System,” American Economic Review, March 1962, pp. 64-103.

2. V. Nemchinov, “Value and Price Under Socialism,” Problems of Economics, IV:3 (July 1961), pp. 3-17.

3. R. Campbell, “Marx, Kantorovich, and Novozhilov: Stolmost Versus Reality,” Slavic Review, October 1961, pp.402-418.

4. M. Bornstein, “The 1963 Soviet Industrial Price Revision,” Soviet Studies, July 1963, pp. 43-52.

5. Supplementary Readings

a) Readings: G. Grossman, “Industrial Prices In the USSR.”

b) A. Wakar and J. Zielinski, “Socialist Operational Price Systems,” American Economic Review, March 1963, pp. 109-127

c) “The Great Value-Price Controversy in the USSR….” in H. Shaffer (ed.), The Soviet Economy (A Collection of Western and Soviet Views), pp. 340-421.

d) A. Zauberman, “Soviet Planometrics,” Soviet Studies, July 1962, pp. 62-74.

E. Finance

1. A. Nove, The Soviet Economy, Chapter 3

2. Readings: F. Holzman, “Financing Soviet Development”

3. Readings: F. Holman, “Soviet Inflationary Pressures, 1928-1957”

4. Readings: D. Hodgman, “Soviet Monetary Controls Through the Banking System”

5. Supplementary Reading. F. Holzman, Soviet Taxation, Chs. 1-4, 9.

F. Labor

1. Readings: E. Brown, “The Soviet Labor Market”

2. Schwartz, pp. 554-565, 534-548

3. Readings: A. Nove, “Social Welfare in the USSR.”

4. W. Galenson, “The Soviet Wage Reform” Proceedings of the Thirteenth Annual Meeting, Industrial Relations Research Association (1961), pp. 250-265.

5. A. Nove, “Toward a Communist Welfare State?” Problems of Communism, January-February 1960.

6. Supplementary Readings

a) E. Nash, “Trends in Labor Controls In the Soviet Union,” in Dimensions, pp. 393-404.

b) E. Brown, “Labor Relations in Soviet Factories,” Industrial and Labor Relations Review, January 1958

c) E. Brown, “A Note on Employment and Unemployment in the Soviet Union in the Light of Technical Progress,” Soviet Studies, January 1961, pp. 231-240.

G. Agriculture

1. Schwarz, Chapter VIII

2. C. Harris, “Soviet Agricultural Resources Reappraised,” Journal of Farm Economics, May 1956, pp. 258-273

3. Readings: L. Volin, “Agricultural Policy of the Soviet Union.”

4. J. Willet, “The Recent Record in Agricultural Production,” in Dimensions, pp. 100-113.

5. H. Walters, Agriculture in the United States and the Soviet Union, U.S. Dept of Agriculture, 1963).

6. Supplementary Readings

a) J. Newth, “Soviet Agriculture: the Private Sector, 1950-59,” Soviet Studies, October 1961 and April 1962.

b) A. Nove, “Soviet Agriculture Marks Time,” Foreign Affairs, July 1962.

c) D. G. Johnson, “Agricultural Production,” in Trends, pp. 214-234

H. Domestic Trade, Foreign Trade and Foreign Aid

1. M. Goldman, Soviet Marketing, Chapters 2, 3, 8.

2. A. Nove, and D. Donnelly, Trade with Communist Countries, pp. 21-57.

3. P. Thunberg, “The Soviet Union in the World Economy,” in Dimensions, pp. 409-438.

4. G. Carnett and M. Crawford, “The Scope and Distribution of Soviet Economic Aid,” in Dimensions, pp. 457-474.

5. A. Zauberman, “Economic Integration,” Problems of Communism, July-August 1959, pp. 23-29.

6. Supplementary Readings

a) M. Goldman, “Product Differentiation and Advertising: Some Lessons from Soviet Experience,” Journal of Political Economy, August 1960, pp. 346-357.

b) Readings: J. Berliner, “Soviet Foreign Economic Competition”

c) Readings: N. Spulber and F. Gehrels, “The Operations of Trade Within the Soviet Bloc.”

d) Readings: F. Holzman, “Some Financial Aspects of Soviet Foreign Trade.”

e) “Discrimination Within the Bloc: Mendershausen vs. Holzman,” The Review of Economics and Statistics: May 1959, May 1960 and May 1962.

I. Future Prospects and Their Implications

1. A. Bergson, “The Great Economic Race: USSR vs. USA,” Challenge, March 1963.

2. A. Bergson and J. Berliner, “Economic Aspects of the Party Program,” The ASTE Bulletin, IV:2 (Winter 1962) pp. 20-36.

3. Readings: O. Hoeffding, “Soviet State Planning and Forced Industrialization as a Model for Asia.”

4. Supplementary Reading

a) J. Hardt et al, The Cold War Economic Gap. (Praeger 1961).

b) Articles by Peterson, Colm, Rostow and Schwartz in Joint Economic Committee, Comparisons of the United States and Soviet Economies, Parts Il and Ill.

Source: Harvard University Archives. Syllabi, course outlines and reading lists in Economics, 1895-2003. Box 8, Folder “Economics, 1963-64”.

_____________________________

HARVARD UNIVERSITY
Department of Economics
Economics 133

Final Examination
January 27, 1964

Part I (One hour)
Answer three of the following four questions.

1. (20 minutes)

Compare the economic policies of Vishnegradsky and Witte with those of Stalin.

2. (20 minutes)

Discuss the problem of “success indicators” in Soviet industry.

3. (20 minutes)

Describe briefly how prices and wages are formed and the role they play in the Soviet economy,

4. (20 minutes)

Describe the operation and role of foreign trade in the Soviet economy.

Part II (One hour)
Answer two of the following three questions,

5. (30 minutes)

“In order to understand why the Russians (at least until very recently) have been so successful in achieving rapid economic growth, one need look no further than the high rate of investment they have been able to attain.”
Discuss.

6. (30 minutes)

You are a high ranking official of the Soviet Communist Party and you have just been appointed chief of agricultural affairs. You are requested to prepare a report in which you are to:

a) describe briefly the major reforms instituted in Soviet agriculture in the last 10 years;
b) discuss the extent of their success and/or failure; and
c) propose some further measures which may be taken to improve the agricultural situation.

How would you respond to this request? (“Flee the country” is not an acceptable response.)

7. (30 minutes)

The U.S. Central Intelligence Agency recently claimed that the rate of growth of Soviet national income in the last two years (1962 and 1963) has been 2.5% per year.
In analyzing this claim, answer the following questions:

a. What is meant by the “index number problem” in measuring economic growth? Why does it arise?

b. To what extent may there be an index number problem in CIA’s calculation?

c. How does this CIA figure differ from the rates of growth of Soviet national income which have been calculated by Bergson and Cohn for earlier periods?

d. What in your opinion might account for the differences?

e. Bonus (If you have time)

What do you think are the prospects for Soviet economic growth in the next 10 to 20 years? Explain.

Part III (One hour)
Answer the following question.

8. (60 minutes)

Suppose the Soviet government were to decide that it were no longer necessary or desirable to maintain a policy of achieving a maximum rate of economic growth. Suppose instead, it were to decide on a policy of achieving moderate growth (say about 4% per year in national income), rising standards of living for its people and full employment (in short, a “civilized” economic policy).

Discuss the possible effects such a decision might have on the various different elements in the organization and operative of the Soviet economy.

Source: Harvard University Archives. Papers Printed for Midyear Examinations [in] History, History of Religions,…, Economics,…,Naval Science, Air Science (January 1964) in bound volume Social Sciences. Final Examinations. January 1964 (HUC 7000.28, vol. 150 of 284).

Image Source: Associate Professor of Economics, Herbert S. Levine. University of Pennsylvania Yearbook, The 1967 Record, p. 108.

Categories
Exam Questions History of Economics Princeton Suggested Reading Syllabus

Princeton. History of Economic Thought. Reading List, General Exam. Baumol, 1987-1988

What I find useful about the syllabus on classical economics from William Baumol’s Princeton course transcribed below is that it provides a lean and precise list of original text reading assignments to work through. Full-blown bibliographies have their use for students when writing term papers, but this butcher’s choice of filet cuts provides a wholesome main course that will last a semester and provide pleasant memories for a lifetime.

Here you will find other postings at Economics in the Rear-view Mirror that offer material from courses in the history of economics.

_______________________

Alan B. Krueger’s Interview with William J. Baumol in Journal of Economic Perspectives, Vol. 15, No. 3 (Summer 2001), pp. 211-231.

_______________________

Princeton University
Department of Economics

Economics 506
History of Economic Thought

Fall Term 1988

Professor W.J. Baumol

Smith, Adam, Wealth of Nations, 1776, Book I, Chapters 1, 2, 3, 5, 6, 7, 8 (first 20 pages), 10 (first 17 pages); Book II, Chapter 3; Book IV, Chapters 1, 2, 8.

Malthus, T.R., An Essay on the Principle of Population, 1798, Pelican Edition, 1970, Introduction by Anthony Flew, Chapters 1-5, 18, 19.

Ricardo, David, On the Principles of Political Economy and Taxation, London, 1817, Chapters I-X, XIX-XXI, XXX-XXXI.

*Ricardo David, Notes on Malthus, Piero Sraffa, editor, Cambridge: Cambridge University Press, 1951, Editor’s Introduction and pp. 300-382.

Marx, Karl and Friedrich Engels, Manifesto of the Communist Party.

Marx, Karl, Capital (3 volumes), New York: International Publishing Company.

Volume I: Author’s Prefaces; Chapter 1; Chapter 3, Sections II a, b; Chapter 5; Chapter 6; Chapter 7, Section 2; Chapter 8; Chapter 9, Sections 1, 3, 4; skim Chapter 10; Chapter 15, Section 6; Chapter 16; Chapter 24, Sections 2, 3, 5; Chapter 25.

Volume II:Preface; Chapter 9; Chapter 16, Part 3; pp. 390-396 (Chapter 17, last 6 pages on Simple Reproduction); Chapter 20; pp. 576-9 (Chapter 21, I, Accumulation in Department 1 (1) formation of a hoard).

Volume III: Preface; Chapters 1, 2, 8, 9, 10, 13, 14, 15, 22, 27, 37, 38, 48.

*Marx, Karl,  A Critique of the Gotha Program, Moscow, Progress Publishers, 1937.

*On reserve at Firestone Library.

Source: Edward Tower (compiler). Economics Reading Lists, Course Outlines, Exams, Puzzles & Problems, Vol. 24. History of Economic Thought. Durham, NC: Eno River Press, August 1990, page 38.

_______________________

PRINCETON UNIVERSITY
Department of Economics

General Examination for the Degree of Doctor of Philosophy

History of Economic Thought
January 1987

Time: 3 hours

  1. Distinguish the roles of the labor command and the labor content discussions of value in Adam Smith. What was Smith’s general model of the determination of long run exchange value under competition?
  2. In terms of the formal Ricardian model, explain the consequences of elimination of a tariff on grain (corn) (a) in the short run; (b) in the long run.
  3. (a) Explain why, in Marx’s view, profits under capitalism can be expected to decline with the passage of time. (b) Why did he reject Ricardo’s model leading to the same conclusion? (c) On what grounds has Marx’s model of the declining profit rate been criticized?
  4. In one sentence each, characterize some of the main work of the following:
    1. Jeremy Bentham
    2. Frederick Bastiat
    3. J.B. Clark
    4. J. R. McCulloch
    5. Enrico Barone
  5. (for Arthur Moretti) Describe the logic of the Hayek business cycle model. What role is played by technological elements? by monetary elements? What is the pertinence of the “Ricardo effect?”
  6. (for Kin Yip Louie) Explain the source of Marshall’s error in using consumers’ surplus to argue that increasing returns industries should be subsidized. Would the Hicksian analysis of the four consumers surpluses have helped ra avoid the error? Why or why not?
  7. (for Susan Skeath [“Susan Skeath van Mulbregt”, Princeton Ph.D., 1989; Professor of Economics at Wellesley College]) Explain the role played by utility in J. S. Mill’s value theory. Does his utility concept lead him to particular policy conclusions? How do Mill’s views on the appropriate role of government differ from those of his classical predecessors?
  8. (for Teow-Hock Koh [“Winston Teow-Hock Koh”, Princeton Ph.D., 1988; Professor at Singapore Management University; died 2013]) (To what extent does Malthus’ contradistinction to his analysis conclusions) structure anticipate the of the Keynesian model? In provide a answering this, summary of the workings of the pertinent parts the Keynesian analysis. What features of Marxian Theory overlap with the Keynesian model?
  9. (for Vicente Morales) a) Describe any of the mathematical solutions to the transformation problem showing how prices and the rate of profit are related to values and the rate of surplus value. b) Explain Samuelson’s criticism of the entire analysis and Morishima’s reply.

Source: Edward Tower (compiler). Economics Reading Lists, Course Outlines, Exams, Puzzles & Problems, Vol. 24. History of Economic Thought. Durham, NC: Eno River Press, August 1990, pp. 266-267.

Image Source:  Cropped from portrait of William J. Baumol in 1981 published in his obituary published in The New York Times, May 10, 2017.

Categories
Chicago Exam Questions Suggested Reading Syllabus

Chicago. Syllabus and Final Exam, International Monetary Economics. Metzler, 1971

The two items below (syllabus and final exam) were incorrectly filed in Lloyd A. Metzler’s papers at Duke. I accidentally stumbled upon both today and thought that rather than trusting my memory of the locations of the syllabus and final exam for Economics 370 in 1971, I’d just transcribe and post the two artifacts today. According to the biographical note below, Lloyd Metzler retired from the University of Chicago in 1971 so this must have indeed been the last time that he taught international monetary economics at Chicago.

__________________________

Biographical Note

Lloyd Appleton Metzler was born on April 3, 1913 in Lost Springs, Kansas. He attended the University of Kansas, where he studied economics under John Ise and earned a Bachelor’s degree in 1935 and an MBA in 1938. Metzler then entered Harvard University. He served as an instructor and tutor at Harvard and completed a Ph.D. in economics in 1942. His dissertation, “Interregional Income Generation,” earned him the Wells Prize. That same year, Metzler was the recipient of a Guggenheim fellowship.

From Harvard, Metzler went on to Washington, D.C., where worked for the Office of Strategic Services and several economic policy and planning commissions between 1943 and 1946. Metzler joined the research staff of the Board of Governors of the Federal Reserve System in 1944. In 1946 he returned to academia when he accepted a teaching position at Yale University. He soon left Yale for the University of Chicago in 1947, where he remained for the rest of his career.

Dr. Metzler survived surgery for a brain tumor in 1952, and with the help of his wife Edith, managed to continue teaching and writing for the next twenty years. He served as Editor of the Journal of Political Economy from 1966 until his retirement in 1971. Metzler made numerous contributions to business cycle literature, macro-monetary theory, tariff theory, mathematical economics, and the field of international trade. The Metzler paradox, Laursen-Metzler effect, and Metzler matrix, all bear his name. He died on October 26, 1980.

Source: University of Chicago Library. Guide to the Lloyd A. Metzler Papers 1941-1948.

__________________________

For Intertemporal Comparison

Syllabus and readings for Economics 370 in 1950.

Exam for Economics 370 in 1953.

Syllabus, readings and final exam for Economics 370 from Winter Quarter 1967.

__________________________

ECONOMICS 370

Monetary Aspects of International Trade
Major Topics and Reading List
Winter, 1971

  1. Mechanism of the Foreign Exchange Market
    1. Alan R. Holmes and Francis Schott, The New York Foreign Exchange Market. New York: The Federal Reserve Bank of New York, 1965, Chapters 1-6.
    2. Frank A. Southard, Jr., Foreign Exchange Practice and Policy. New York: The McGraw-Hill Book Company, 1940.
    3. Norman Crump, The ABC of the Foreign Exchange. London: MacMillan and Company, Ltd., 1951.
    4. James E. Meade, The Theory of International Economic Policy: Vol. I. The Balance of Payments. London: Oxford University Press, 1951, Chapter 1.
  2. The Quantity of Money, the Rate of Interest, and the Price Level
    1. James Tobin, “The Monetary Policy and the Management of the Public Debt: The Patman Inquiry,” Review of Economics and Statistics, Vol. XXXV, No. 2, May 1953, pp. 118-27.
    2. Subcommittee on General Credit Control and Debt Management of the Joint Committee on the Economic Report, Hearings on the Question, What Should our Monetary and Debt Management Policy Be? 82nd Congress of the United States 1952, pp. 688-711, 691-98. (These pages include the testimony of Milton Friedman and Paul Samuelson).
    3. Robert V. Roosa, “Interest Rates and the Central Bank,” In Money, Trade, and Economic Growth, in honor of John Henry Williams, New York: The Macmillan Company, 1951.
    4. Lloyd A. Metzler, “Wealth, Saving, and the Rate of Interest,” Journal of Political Economy, Vol. LIX, No. 2, April 1951, pp. 93-116.
    5. Robert A. Mundell, “The Public Debt, Corporate Income Taxes, and the Rate of Interest,” Journal of Political Economy, Vol. LXVIII, No. 6, December 1960, pp. 622-26.
    6. George Horwich, “Real Assets and the Theory of Interest,” Journal of Political Economy, Vol. LXX, No.2, April 1962, pp. 158-69.
    7. Don Patinkin, Money, Interest, and Prices, 1st edition, Evanston: Row, Peterson and Company, 1956, Part II.
  3. The Role of Money in International Adjustment: Full Employment and Under-Employment
    1. J. M. Keynes, Treatise on Money: Vol. 1. The Pure Theory of Money. London: Macmillan and Company, 1935, Chapter 21.
    2. Lloyd A. Metzler, “The Theory of International Trade,” from A Survey of Contemporary Economics, Howard S. Ellis, Editor, Homewood, Ill.: R.D. Irwin, Inc., 1948.
    3. Lloyd A. Metzler, “The Process of International Adjustment Under Conditions of Full Employment: A Keynesian View,” first delivered before the Econometric Society, 1960, republished in AEA, Readings in International Economics, Vol. XI, Richard Caves and Harry Johnson, editors, Homewood, Ill.: R. D. Irwin, Inc., 1968, Chapter 28.
  4. Free-Market Exchange Rates
    1. A. J. Brown, “The Foreign Exchanges,” in Oxford Studies in the Price Mechanism, T. Wilson and P.W.S. Andrews, editors, Oxford at the Clarendon Press, 1951, Part II.
    2. Sidney Alexander, “Effects of a Devaluation on a Trade Balance,” International Monetary Fund Staff Papers, Vol. Il, No. 2, April 1952.
    3. Milton Friedman, “The Case for Flexible Exchange Rates,” in Essays in Positive Economics, Chicago: The University of Chicago Press, 1953, pp. 157-203.
    4. Joan Robinson, “The Foreign Exchanges,” in Essays in the Theory of Employment. Oxford: Basil Blackwell, 1947, Part III.
    5. Lloyd A. Metzler, “Exchange Rates and the International Monetary Fund,” in International Monetary Policies, Postwar Economic Studies, No. 7, Washington, D.C.: The Board of Governors of the Federal Reserve System, September 1947.
    6. Rudolph R. Rhomberg. “A Model of the Canadian Economy under Fixed and Fluctuating Exchange Rates,” Journal of Political Economy, Vol. LXXII, No. 1, February 1964, pp. 1-31.
  5. Forward Exchange Rates
    1. Paul Einzig, The Theory of Forward Exchange. London: Macmillan and Company, Ltd., 1937.
    2. Paul Einzig, A Dynamic Theory of Forward Exchang. London: Macmillan and Company, New York: St. Martin’s Press, 1961.
    3. Alan R. Holmes and Francis Schott, The New York Foreign Exchange Market. New York: The Federal Reserve Bank of New York, 1965, Chapters 7-8.
    4. Paul Einzig, “Some Recent Development in Official Forward Exchange Operations,” Economic Journal, Vol. LXXIII, No. 290, June 1963, pp. 241-53.
    5. Paul Einzig, “Some Recent Changes in Forward Exchange Practices,” Economic Journal, Vol. LXX, No. 279, September 1960, pp. 485-95.
  6. The Balance of Payments and the Concepts of Income
    1. R. F. Bennett, “Significance of International Transactions in National Income,” in Studies in Income and Wealth, Vol. VI. New York: The National Bureau of Economic Research, 1943.
    2. U. S. Department of Commerce, Income and Output, 1958 Supplement to the Survey of Current Business.
  7. The Theory of Income Transfers
    1. J. M. Keynes, “The Transfer Problem,” Economic Journal, Vol. XXXIX, No. 153, March 1929, pp. 1-7.
    2. B. Ohlin, “The Reparation Problem: A Discussion. I. Transfer Difficulties, Real and Imagined,” Economic Journal, Vol. XXXIX, No. 154, June 1929, pp. 172-78.
    3. J. M. Keynes. “The Reparation Problem: A Discussion. II. A Rejoinder,” Economic Journal, Vol. XXXIX, No. 154, June 1929, 179-82.
    4. J. Rueff, “Mr. Keynes’ Views on the Transfer Problem,” Economic Journal, Vol. XXXIX, No. 155, September 1929, pp. 388-99.
    5. B. Ohlin, “Rejoinder to J. Rueff,” Economic Journal, Vol. XXXIX, No. 155, September 1929, pp. 400-04.
    6. J. M. Keynes, “Reply to J. Rueff,” Economic Journal, Vol. XXXIX, No. 155, September 1929, pp. 404-08.
    7. L. A. Metzler, “The Transfer Problem Reconsidered,” Journal of Political Economy, Vol. L, No. 3, June 1942, pp. 397-414.
    8. H. G. Johnson, “The Transfer Problem and Exchange Stability,” Journal of Political Economy, LXIV, No. 3, June 1956, pp. 212-25, Republished in International Trade and Economic Growth. London: George Allen and Unwin, Ltd., 1958, Chapter VII.
    9. L. A. Metzler, “Flexible Exchange Rates, The Transfer Problem, and the Balanced-Budget Theorem,” Rivista Internazionale di Scienze Economiche e Commerciale, Anno XIII, No. 4, April 1966, pp. 301-18. Republished in Essays in Honor of Marco Fanno, Vol. II, Tullio Bagiotti, editor, edizioni cedam, Padova, 1966, pp. 449-76.
  8. Evolution of the International Monetary System
    1. Randall Hinshaw. Toward Currency Convertibility. Princeton University, Essays in International Finance, No. 31, 1958.
    2. Robert Triffin, Europe and the Money Muddle, New Haven: Yale University Press, 1957.
    3. Charles P. Kindleberger. The Dollar Shortage, Cambridge, Mass.: The Technology Press, New York: John Wiley and Sons, Inc., 1950.
    4. Robert Triffin, “The International Monetary Position of the United States,” in The Dollar in Crisis, Seymour E. Harris, editor, New York: Harcourt, Brace and World, Inc., 1961.
    5. Hal B. Lary, Problems of the United States as World Trader and Banker, Princeton: Princeton University Press for the National Bureau of Economic Research, 1963.
    6. Robert Triffin, The Evolution of the International Monetary System: Historical Reappraisal and Future Perspectives. Princeton Studies in International Finance Section. Princeton University, 1964.

Source: Duke University. David M. Rubenstein Rare Book and Manuscript Library. Economists’ Papers Archive. Lloyd Appleton Metzler Papers, Box 9, Folder “The Dust Proof File”.

__________________________

L. A. Metzler

ECONOMICS 370
FINAL EXAMINATION
WINTER, 1971

Answer all questions

  1. It is frequently said that forward purchases and sales have a two-fold effect upon the assets and liabilities of the bank making the forward transactions. When a bank makes a forward purchase, for example, it is entitled to receive a given amount of foreign currency some time in the future and this right to receive foreign currency constitutes an asset. On the other hand, when the bank receives the foreign currency, it is obligated to pay a given amount of its own currency, and exchange, and this obligation constitutes a liability. Conversely, when the bank makes a forward sale, it is obligated to deliver a given amount of foreign currency some time in the future and such an obligation constitutes a liability. But when the bank delivers the foreign currency, it is entitled to receive a given quantity of domestic currency to complete the transaction, and this receipt constitutes an asset.
    1. In view of the two-fold effect of both forward sales and purchases, how can you justify the inclusion of forward sales as liabilities and the inclusion of forward purchases as assets in an account of the bank’s foreign currency position?
    2. If it is actually true that both sales and purchases increase assets and liabilities by the same amount, what does this imply with respect to the bank’s ability to put itself in a closed position by means of operations on the forward exchange market?
  2. The table, below, presents the yield on 90-day Canadian treasury bills (rc), the yield on 90-day U.S. treasury bills (rus), the 90-day forward exchange rate (FR), and the spot exchange rate (SR) for Canadian currency. The yields for both Canadian and United States bills are stated on an annual basis and the exchange rates represent the price, in United States dollars, of one unit of Canadian dollars. The table covers various periods of time, from A to I.
Period of time Yield on
90-day Canadian treasury bills (rc)
Yield on
90-day
U.S.
treasury bills (rus)
90-day forward exchange rate (FR) Spot exchange rate (SR)
A .05 .06 1.0100 1.000
B .06 .05 1.0050 1.000
C .06 .04 1.0025 1.000
D .07 .03 0.9975 1.000
E .05 .04 0.9900 1.000
F .04 .05 0.9950 1.000
G .02 .04 0.9975 1.000
H .02 .06 1.0025 1.000
I .01 .05 2.0050 2.000

On the basis of the information given in the table above, you are asked to:

    1. Indicate which periods are periods of short-term potential capital outflow (O), and which periods are periods of short-term capital inflow (I) from the point of view of U.S. banks.
    2. Prepare a table showing the profits or losses on security transactions, the profits or losses on currency transactions, and the net outflow or inflow margin for all time intervals from A to I.
    3. Show what transactions a U.S. bank would make in carrying out a short term covered capital outflow and what transactions the bank would make in carrying out a short term capital inflow. From this information, show that arbitrage activities of the U.S. bank always influence rc, rus, FR, and SR in such a way as to put the market values back on the interest parity line.
    4. Comment on the stabilizing effects of interest arbitrage.
  1. Professor Alexander maintains that devaluation will be ineffective unless fiscal measures are taken to control spending.
    1. Discuss Alexander’s argument.
    2. Is it also applicable against a system of flexible exchange rates? Explain.
  2. Country A and Country B are trading with each other under a system of flexible exchange rates. Country A makes a unilateral transfer of t currency units, payable in the currency of B. On the assumption that both countries balance their budgets, prove that factor income remains unchanged in both countries while net output rises in A and falls in B. Give both an algebraic and a commonsense explanation of these results. What bearing do these results have on the controversy between Keynes and Ohlin concerning German reparations?
  3. The success of the Canadian experiment with flexible exchange rates is frequently given as an argument for the introduction of flexible exchange rates as a means of eliminating the deficit in the U.S. Balance of Payments.
    1. Would Alexander’s absorption principle apply to this situation?
    2. Can you see any reason why the comparison between Canada and the United States might be inappropriate?

Source: Duke University. David M. Rubenstein Rare Book and Manuscript Library. Economists’ Papers Archive. Lloyd Appleton Metzler Papers, Box 9, Folder “Econ 371 [sic] Reading List.”

Source Image: Posting by Margie Metzler on the Metzler Family Tree at the genealogical website, ancestry.com.